[obm-l] RE: [obm-l] Questão de probabilidade(dúvida sobre gabarito)

2011-10-06 Por tôpico Antonio Neto

Certamente o primeiro homem pode sentar em 8 lugares. Mas o segundo só tem 3, e 
o terceiro 2, e o quarto 1. Faça a figura. e 4! para as mulheress, o que dá 
0,0285714..., e a resposta é letra A.


Antonio Olavo da Silva Neto
 




From: marconeborge...@hotmail.com
To: obm-l@mat.puc-rio.br
Subject: [obm-l] Questão de probabilidade(dúvida sobre gabarito)
Date: Sun, 25 Sep 2011 05:14:44 +





Um grupo de pessoas,composto por 4 homens e 4 mulheres,compra 8 cadeiras 
consecutivas na mesma fila de um teatro.Se eles se sentarem 
aleatóriamente,nessas cadeiras,a probabilidade de que homens e mulheres se 
sentem em cadeiras alternadas é aproximadamente:
 
a) 2,86%  b) 5,71%   c) 1,43%d) 11,42% 
 
 O primeiro homem pode sentar em 8 lugares.O segundo pode sentar em 6 lugares.O 
terceiro,em 4 lugares e o quarto,em 2 lugares.Como sobram 4 lugares,é só 
permutar as 4 mulheres.Então o número de possibilidades  de que homens e 
mulheres sentem em cadeiras alternadas é 8x6x4x2x4x3x2x1.Dividindo esse número 
por 8!(que é o total de possibilidades) temos 5/35 = 0,2285.
O que me intriga é que esse resultado é o dobro dos 11,42% do item d
Alguem poderia esclarecer?
  

[obm-l] Divisibilidade por 13

2010-12-20 Por tôpico Antonio Neto

   Senhores, permitam meter a colher torta. Com a mesma notação do texto, um 
outro possível critério é: n = 10x + a é divisível por 13 se, e somente se, x + 
4a o for. Note que vc multiplica o algarismo final por -9, e eu por 4. Ahá!!! 
4-(-9) = 13. Experimente também x + 17a, etc... Há um livrinho russo, da 
Editora Mir, o exemplar que tenho está em espanhol, chamado Criterios de 
divisibilidad, acho que é do Vorobiov, mas não estou em casa agora. 
Divirta-se, abraços, olavo.


Antonio Olavo da Silva Neto


 



Date: Fri, 17 Dec 2010 11:54:57 -0200
From: ne...@infolink.com.br
To: obm-l@mat.puc-rio.br
Subject: Re: [obm-l] Divisibilidade por 13


Oi, Felipe,

Você vai gostar de 
http://www.egge.net/~savory/maths1.htm

Seu caso é equivalente ao que o texto menciona. Procure perceber isto.

Abraços,
Nehab


Em 16/12/2010 23:55, Felipe Diniz escreveu: 
n = 10x+a, a entre 0 e 9. 


x-9a = 0 mod13
entao x=9a mod13


n= 10x+a = 91a = 13*7a = 0  mod 13


2010/12/16 marcone augusto araújo borges marconeborge...@hotmail.com


Dado um número,8281,por exemplo.Fazendo 828 - 9*1=819 e repetindo o 
procedimento:81 - 9*9=0
zero é divisível por 13,logo8281 também é.
Para 867:86 - 9*7=23.
23 não é divisível por 13,logo 867 também não é.
Como provar que a regra é verdadeira?
 


  

[obm-l] Dízima periódica

2010-10-20 Por tôpico Antonio Neto

   Jovem, comecemos pela base 10. Uma fracao, na base 10, gera uma dízima 
simples se o seu denominador for primo com dez, ou seja, se nao contiver 
fatores primos nem 2 nem 5. Se contiver fatores 2 e/ou 5 e mais algum primo, 
gera uma dizima composta, eh o caso do seu exemplo. E se so contiver fatores 2 
e/ou 5, teremos um decimal finito. Desculpe nao mandar a bibliografia, estou na 
casa do meu filho, sem acesso a minha bisbilhoteca. Se nao me engano, o livro 
de analise do Djairo Figueiredo tem a prova disso.
 
   Analogamente, a sua fracao eh 17/6, cujos fatores primos sao 2 e 3. Logo, 
teremos uma dizima simples se a base nao contiver fatores 2 e 3, ou seja, 5 e 
7. Uma composta se contiver fator 2 e/ou 3 e mais algum outro primo, o que quer 
dizer ninguem menor que 10.
 
   Espero haver contribuido. Saudacoes periodicas de mim, olavo.


Antonio Olavo da Silva Neto


 



Date: Mon, 18 Oct 2010 14:58:09 -0200
Subject: [obm-l] Re: [obm-l] Dízima periódica
From: aadornell...@gmail.com
To: obm-l@mat.puc-rio.br

6?


2010/10/18 Pedro Chaves brped...@hotmail.com





A fração, na base dez, 17/6 não gera uma dízima periódica se mudarmos para que 
base de numeração menor do que dez?

  

[obm-l] Nosso calendario

2010-09-16 Por tôpico Antonio Neto

   Desculpem meter a colher torta em uma questao ja resolvida. Eh uma linda 
questao, e eu sempre comecava o meu curso de combinatoria com ela, para alunos 
do segundo ano do ensino medio. Como eles nao sao familiarizados com 
congruencias, eu utilizava a solucao do Niven - Ivan Niven, Mathematics of 
Choice, MAA (Mathematics Association of America, a minha edicao eh da decada de 
70, deve haver uma mais recente). Eh linda, na verdade eh a mesma de voces, mas 
ele consegue evitar falar de congruencias com rara elegancia. Agora, que o 
cachorro morto ja foi devidamente espancado, so resta deixar osculos e amplexos 
de mim, olavo.


Antonio Olavo da Silva Neto


 



Date: Sun, 29 Aug 2010 10:32:59 -0300
Subject: Re: [obm-l] FW: Nosso calendario
From: msbro...@gmail.com
To: obm-l@mat.puc-rio.br

Sim, apesar de ser imediato.
Pois fevereiro ganha 1 dia.. logo, basta somar 1 nos devidos locais e ver que 
ainda
temos todos os resíduos módulo 7.


abraços,
Salhab



2010/8/29 Hugo Fernando Marques Fernandes hfernande...@gmail.com

Não faltou considerar os anos bissextos?

Abraços.

Hugo.


Em 29 de agosto de 2010 00:26, marcone augusto araújo borges 
marconeborge...@hotmail.com escreveu:





Obrigado,abraços.
 


Date: Sat, 28 Aug 2010 23:41:47 -0300
Subject: Re: [obm-l] FW: Nosso calendario
From: msbro...@gmail.com



To: obm-l@mat.puc-rio.br


Vamos ver a qtde de dias de cada mês, em ordem:
31, 28, 31, 30, 31, 30, 31, 31, 30, 31, 30, 31


Analisando isso módulo 7, visto que são 7 dias da semana, temos:
28 == 0 (mod 7)
30 == 2 (mod 7)
31 == 3 (mod 7)


Desta maneira, temos:
3, 0, 3, 2, 3, 2, 3, 3, 2, 3, 2, 3


Supondo que o primeiro dia 13 esteja em k, temos:
k, k+3, k+3, k+6, k+8, k+11, k+13, k+16, k+19, k+21, k+24, k+26


Analisando mod 7, temos:
k, k+3, k+3, k+6, k+1, k+4, k+6, k+2, k+5, k, k+3, k+5


Veja que temos todos os inteiros mod 7 somando com k.
Desta maneira, sempre há uma sexta feira 13.


abraços,
Salhab






2010/8/28 marcone augusto araújo borges marconeborge...@hotmail.com



 


From: marconeborge...@hotmail.com
To: obm-l@mat.puc-rio.br
Subject: Nosso calendario
Date: Sun, 29 Aug 2010 02:12:05 +

Mostre q em qualquer ano existe pelo menos uma sexta-feira 13.Eu acho q 
consegui resolver,mas gostaria de ver outra solução.
Fiz assim:se o dia 13 de janeiro  é um domingo,entao o dia 13 de setembro é uma 
sexta pois,contando apenas o numero 
de dias q passam de 28 em cada mes,a partir de janeiro(até agosto),encontramos 
19 dias(um multiplo de 7 mais 5),dai,conside-
rando o domingo como dia 1,temos 1+5=6(sexta).Usei o mesmo raciocinio para o 
caso do dia 13 de janeiro ser segunda,terça,quarta,quinta ou sabado e encontrei 
para cada caso uma sexta feira 13 no mesmo ano.



  

[obm-l] CONVITE

2009-11-11 Por tôpico Antonio Neto











CONVITE


Oi, tudo bem?
Estou lhe mandando o convite, e todos os detalhes (endereço, horário e etc...) 
estão no anexo.


1º Obs: Favor imprimir o convite anexado a esse E-Mail, pois é indispensável a 
apresentação do mesmo.
2º Obs: Convite com direito a acompanhante.
3º Obs: Só sera permitida a entrada com o convite em mãos.
4º Obs: Não deixar de ir...


Abraços!

 1 anexo
Convite.doc (0,7 KB)
Visualizar , Baixar ou Imprimir 



  
_
Keep your friends updated—even when you’re not signed in.
http://www.microsoft.com/middleeast/windows/windowslive/see-it-in-action/social-network-basics.aspx?ocid=PID23461::T:WLMTAGL:ON:WL:en-xm:SI_SB_5:092010

[obm-l] CONVITE

2009-10-27 Por tôpico Antonio Neto











CONVITE


Oi, tudo bem?
Estou lhe mandando o convite, e todos os detalhes (endereço, horário e etc...) 
estão no anexo.


1º Obs: Favor imprimir o convite anexado a esse E-Mail, pois é indispensável a 
apresentação do mesmo.
2º Obs: Convite com direito a acompanhante.
3º Obs: Só sera permitida a entrada com o convite em mãos.
4º Obs: Não deixar de ir...


Abraços!

 1 anexo
Convite.doc (0,7 KB)
Visualizar , Baixar ou Imprimir 



  
_
Windows Live: Make it easier for your friends to see what you’re up to on 
Facebook.
http://www.microsoft.com/middleeast/windows/windowslive/see-it-in-action/social-network-basics.aspx?ocid=PID23461::T:WLMTAGL:ON:WL:en-xm:SI_SB_2:092009

[obm-l] CONVITE

2009-10-21 Por tôpico Antonio Neto











CONVITE


Oi, tudo bem?
Estou lhe mandando o convite, e todos os detalhes (endereço, horário e etc...) 
estão no anexo.


1º Obs: Favor imprimir o convite anexado a esse E-Mail, pois é indispensável a 
apresentação do mesmo.
2º Obs: Convite com direito a acompanhante.
3º Obs: Só sera permitida a entrada com o convite em mãos.
4º Obs: Não deixar de ir...


Abraços!

 1 anexo
Convite.doc (0,7 KB)
Visualizar , Baixar ou Imprimir 



  
_
Windows Live: Friends get your Flickr, Yelp, and Digg updates when they e-mail 
you.
http://www.microsoft.com/middleeast/windows/windowslive/see-it-in-action/social-network-basics.aspx?ocid=PID23461::T:WLMTAGL:ON:WL:en-xm:SI_SB_3:092010

[obm-l] CONVITE

2009-10-19 Por tôpico Antonio Neto











CONVITE


Oi, tudo bem?
Estou lhe mandando o convite, e todos os detalhes (endereço, horário e etc...) 
estão no anexo.


1º Obs: Favor imprimir o convite anexado a esse E-Mail, pois é indispensável a 
apresentação do mesmo.
2º Obs: Convite com direito a acompanhante.
3º Obs: Só sera permitida a entrada com o convite em mãos.
4º Obs: Não deixar de ir...


Abraços!

 1 anexo
Convite.doc (0,7 KB)
Visualizar , Baixar ou Imprimir 



  
_
Windows Live: Make it easier for your friends to see what you’re up to on 
Facebook.
http://www.microsoft.com/middleeast/windows/windowslive/see-it-in-action/social-network-basics.aspx?ocid=PID23461::T:WLMTAGL:ON:WL:en-xm:SI_SB_2:092009

[obm-l] CONVITE

2009-10-17 Por tôpico Antonio Neto











CONVITE


Oi, tudo bem?
Estou lhe mandando o convite, e todos os detalhes (endereço, horário e etc...) 
estão no anexo.


1º Obs: Favor imprimir o convite anexado a esse E-Mail, pois é indispensável a 
apresentação do mesmo.
2º Obs: Convite com direito a acompanhante.
3º Obs: Só sera permitida a entrada com o convite em mãos.
4º Obs: Não deixar de ir...


Abraços!

 1 anexo
Convite.doc (0,7 KB)
Visualizar , Baixar ou Imprimir 



  
_
Windows Live: Make it easier for your friends to see what you’re up to on 
Facebook.
http://www.microsoft.com/middleeast/windows/windowslive/see-it-in-action/social-network-basics.aspx?ocid=PID23461::T:WLMTAGL:ON:WL:en-xm:SI_SB_2:092009

[obm-l] Combinatória

2009-03-08 Por tôpico Antonio Neto

   Amigo Wagner,

 

   receio que a solucao nao esteja totalmente correta, havendo contagem dupla. 
A questao pede dois conjuntos, não os havendo sequer nomeado (nada de dois 
conjuntos A e B, por exemplo). Eu sugeriria C(8, 4) = 70, pelo seguinte 
raciocinio: vamos colocar o 1 em um deles e o 8 no outro. Selecione 4 entre os 
8 restantes e os coloque no que contem o 1. O outro conjunto se completa 
automaticamente.

 

   Desculpe se fui meio conciso demais. Uma boa ideia para explicar aos alunos 
eh reduzir a um caso mais simples, com os numeros de 1 a 6, em dois conjuntos 
de tres, estando o 1 e o 2 em conjuntos separados. Sai no tapa rapidinho, e 
eles enxergam a duplicacao do outro raciocinio.

 

Espero ter ajudado, amplexos de mim, olavo.


Antonio Olavo da Silva Neto


 



Date: Mon, 22 Dec 2008 13:32:54 -0200
From: wtade...@gmail.com
To: obm-l@mat.puc-rio.br
Subject: [obm-l] Combinatória de Natal


Amigos,
 
Separam-se os números imteiros de 1 a 10 em dois conjuntos de 5 elementos, de 
modo que 1 e 8 não fiquem no mesmo conjunto. de quantas maneiras isso pode ser 
feito? 
Pensei assim: 
a) Fixando o 1 no 1º grupo teria C(8,4) formas de escolher os 4 restantes sem 
contar com o 8.
Esse resultado multiplicado por C(4,4) formas de montar o outro grupo com o 8.
b) Como o 1 poderia estar no 2º grupo, o reusltado final seria:
2xC(8,4)xC(4,4)=2 x 70 x 1 = 140.
Pensei certo?
 
Abraços
-- 
Walter Tadeu Nogueira da Silveira


_
Show them the way! Add maps and directions to your party invites. 
http://www.microsoft.com/windows/windowslive/products/events.aspx

RE: [obm-l] Contagem

2008-11-22 Por tôpico Antonio Neto

Oi,
receio que haja alguns pequenos enganos. No caso PPP, tudo bem, mas o outro 
caso nao eh PPI, mas PII, o que nao acarretaria problemas de contas se tivesse 
sido resolvido corretamente. Ele se divide em tres casos, PII, PIP e IPP, logo 
o seu 50 eh na verdade 50*3 = 150. Acho que agora estah tudo certinho. 
Amplexos, olavo
Antonio Olavo da Silva Neto

Date: Fri, 21 Nov 2008 20:22:26 -0200From: [EMAIL PROTECTED]: [EMAIL 
PROTECTED]: [obm-l] Contagem
O problema abaixo foi trazido por um aluno. Eis a solução encontrada pela turma:
 
O número de possibilidades de escolha de 3 números naturais distintos de 1 a 
10, de modo que sua soma seja sempre par, é:

120 
220 
150 
290 
160 
SOLUÇÃO. Supõe-se que são cartões com os números onde:
Pares: 2, 4, 6, 8 e 10
Ímpares: 1, 3, 5, 7, 9
Para que a escolha dos três números dê soma par, deve-se ter: P P P ou I P P
a) P P P temos: C(5,3) = 10
b) I P P temos: C(5,1) x C(5,2) = 5 x 10 = 50
Total de 10 + 50 = 60 possibilidades.
Ficaram felizes, mas a resposta apontava 160. Não consegui mostrar o erro a 
eles. Alguém poderia dar uma ajuda? Grato.Walter Tadeu Nogueira da Silveira
_
News, entertainment and everything you care about at Live.com. Get it now!
http://www.live.com/getstarted.aspx

RE: [obm-l] Menor ângulo

2008-11-07 Por tôpico Antonio Neto

   Oi, acredito em duendes e errinhos de digitacao. Nao seria sqrt(2) = 1,414? 
Ai os lados medem 1000sqrt(2) e 1000sqrt(3). Ve se ajuda. Amplexos, olavo.
Antonio Olavo da Silva Neto

From: [EMAIL PROTECTED]: [EMAIL PROTECTED]: [obm-l] Menor ânguloDate: Wed, 5 
Nov 2008 14:39:54 +

 Uma ajudinha por favor: 1) Três estradas retilíneas devem conectar os pontos 
A, B e C. Sabe-se que a distância entre A e B é igual a 1414m, que o ângulo ACB 
deve ser reto e que o comprimento total do percurso ACB deve ser de 1732m. 
Nestas condições, considerando sqrt[2]=1,4114 e sqrt[3]=1,732, o menor ângulo 
BAC possível deve medir, graus, exatamente a) 15   b)10   c) 5   d) 20   e)30 
Observação:  Sqrt[n] - raiz quadrada de n  

Notícias direto do New York Times, gols do Lance, videocassetadas e muitos 
outros vídeos no MSN Videos! Confira já!
_
Discover the new Windows Vista
http://search.msn.com/results.aspx?q=windows+vistamkt=en-USform=QBRE

[obm-l] Probabilidade - Caixa sem Troco

2008-10-24 Por tôpico Antonio Neto

   Oi, Luiz,
   isto eh o chamado Principio da Reflexao, e eh facil encontra-lo no livo do 
Morgado de Combinatoria, editado pelo IMPA. Nao olhei, mas certamente estarah 
no Wolfram.
   Amplexos, olavo.
Antonio Olavo da Silva Neto

Date: Tue, 21 Oct 2008 09:33:36 -0700From: [EMAIL PROTECTED]: [obm-l] 
Probabilidade - Caixa sem Troco To: obm-l@mat.puc-rio.br






Uma determinada atração custa R$ 5. Temos 2n pessoas em fila, sendo que n 
possuem uma nota de R$ 5 e as outras n possuem uma nota de R$ 10. Qual a 
probabilidade desta fila dar problema ? (ou seja, o caixa ficar sem troco em um 
dado momento)
 
 
Abs
Felipe.

Novos endereços, o Yahoo! que você conhece. Crie um email novo com a sua cara 
@ymail.com ou @rocketmail.com.
_
Connect to the next generation of MSN Messenger 
http://imagine-msn.com/messenger/launch80/default.aspx?locale=en-ussource=wlmailtagline

RE: [obm-l] Combinatoria e Prob

2008-10-24 Por tôpico Antonio Neto

   Oi, Thais,
   o numero procurado no seu segundo problema eh o de permutacoes caoticas 
(derangements) de n objetos. Alem das referencias classicas da internet, estah 
no livro do Morgado, publicado pelo IMPA, no Mathematics of Choice, do Niven, 
publicado pela MAA, e, para os dinossauros como eu, no livro Preludio aa 
Analise Combinatoria, Ed. Nacional, esgotado, mas quem sabe uma vasculhada em 
sebos e bisbilhotecas...Amplexos de mim, olavo.
Antonio Olavo da Silva Neto
_
Discover the new Windows Vista
http://search.msn.com/results.aspx?q=windows+vistamkt=en-USform=QBRE

RE: [obm-l] Mais uma de análise combinatória

2008-10-20 Por tôpico Antonio Neto


Antonio Olavo da Silva Neto   Peço notar que hah consoantes e vogais repetidas, 
e isto deve ser levado em vonta. Nem sempre eh fazer um caso e multiplicar por 
dois. E com ASSOMBRACAO, desprezando cedilha e til?

Date: Mon, 6 Oct 2008 15:17:55 -0300From: [EMAIL PROTECTED]: [EMAIL PROTECTED]: 
Re: [obm-l] Mais uma de análise combinatória
Pensa que os anagramas terão que ter consoantes e vogais alternadas. Como o 
número de consoantes é igual ao número de vogais, então pode-se começar com 
consoante ou vogal.É só fazer um dos casos e multiplicar por 2.
2008/10/6 Marcelo Costa [EMAIL PROTECTED]

Peço perdão, pois enviei a questão incompleta, faltou o que está me gerando as 
dúvidas.Quantos são os anagramas da palavra ENGENHARIA os quais não possuem 
vogais juntas.
_
Connect to the next generation of MSN Messenger 
http://imagine-msn.com/messenger/launch80/default.aspx?locale=en-ussource=wlmailtagline

[obm-l] Uma ajuda

2008-10-09 Por tôpico Antonio Neto

   Ve se ajuda: coloque primeiro as consoantes, o que pode ser feito de 5.4 = 
20 modos. Para as vogais, já que são diferentes, soh hah um modo, e a resposta 
eh 20. A ideia eh que se algo deve ser colocado em ordem (crescente, 
alfabetica, etc,...) coloque primeiro os outros elementos. Amplexos, olavo.
Antonio Olavo da Silva Neto

Date: Tue, 7 Oct 2008 14:24:24 -0300From: [EMAIL PROTECTED]: [EMAIL PROTECTED]: 
[obm-l] Uma ajuda

Amigos,Quantos anagramas da palavra ALUNO mantém as vogais e ordem alfabética?
 
Vejam se concordam para explicar aos alunos.
 
i) A pode ocupar a 1ª, 2ª ou 3ª casa somente. 
ii) O pode ocupar 2ª, 3ª ou 4ª casa.
iii) U pode ocupar 3ª, 4ª ou 5ª.
A resposta é 20. Mas queria construir uma solução mais clara. Alguma idéia?
 
Abraços-- 
Walter
_
News, entertainment and everything you care about at Live.com. Get it now!
http://www.live.com/getstarted.aspx

[obm-l] ...fórmula da mediana????

2008-10-07 Por tôpico Antonio Neto

   Caro Nehab, não quis chamar ninguem de velho, apenas observei que o 
exercicio podia ser resolvido usando coisinhas mais simples. Ateh porque em 
questao de idade nao posso ser muito exigente. Desculpe se houve alguma ofensa, 
embora nao seja do Rio, jah o conhecia desde Salvador, de onde vim, pra ver se 
aprendia Matematica. Ainda nao consegui ateh hoje, mas a minha admiracao por 
voce continua a mesma. Abracos respeitosos de mim, olavo.
Antonio Olavo da Silva Neto

Date: Fri, 3 Oct 2008 17:07:23 -0300From: [EMAIL PROTECTED]: [EMAIL PROTECTED]; 
[EMAIL PROTECTED]: [Fwd: [obm-l] geometria...fórmula da mediana]


Não quero fofocar não, mas tão chamando neguinho de meia idade de velho... Mas 
qual a culpa dos referidos hoje senhores se nos livros em que eles estudaram, 
além de zilhões de exercícios ótimos para resolver, adoravam dar nomes óbvios 
às formuletas?   Alguém por ai, com menos de 20 anos sabe, AGORA, sem colar e 
de cor, formuletas para cálculo das bissetrizes internas e externas de um 
triângulo?  Duvi-dê-ó-dó...  Oh, o vestiba tá rolando.  E como calcular o 
volume de um barril de chopp?  Então, Olavo, tome de fórmuleta da mediana 
procê...Nehab :-P  Mensagem original  



Assunto: 
[obm-l] geometria...fórmula da mediana

Data: 
Fri, 3 Oct 2008 18:49:02 +

De: 
Antonio Neto [EMAIL PROTECTED]

Responder a: 
obm-l@mat.puc-rio.br

Para: 
obm-l@mat.puc-rio.br

Referências: 
[EMAIL PROTECTED]

   Oi, acontece que as unicas pessoas que sabem essas formulas sao o Wagner e o 
Paulo Cesar. Por acaso sei deduzi-las, mas vamos mais devagar. Seja G o 
baricentro. Ponha GE = x e BC = 2x. Do mesmo modo, GD = y e GB = 2y. Use 
Pitagoras nos triangulos BEG e GCD, some tudo e seja o que Deus quiser. Se nao 
der certo, mas vai dar, eh soh escrever. Amplexos, olavo.
Antonio Olavo da Silva Neto

  

Seja ABC um triângulo de lados BC, CA, AB cujas medidas sãorespectivamente 
iguais a a, b, c. Se D e E são os pontos médios de AC e ABrespectivamente, 
mostre que a mediana BD é perpendicular a CE se, e somentese, b² + c² = 5a² 
 
 
É suficente usar a fórmula que fornece a mediana?
 
Grato
 
 
 
 
 

Explore the seven wonders of the world Learn more! 
= 
Instruções para entrar na lista, sair da lista e usar a lista em 
http://www.mat.puc-rio.br/~obmlistas/obm-l.html 
= 
_
News, entertainment and everything you care about at Live.com. Get it now!
http://www.live.com/getstarted.aspx

[obm-l] geometria...fórmula da mediana????

2008-10-03 Por tôpico Antonio Neto

   Oi, acontece que as unicas pessoas que sabem essas formulas sao o Wagner e o 
Paulo Cesar. Por acaso sei deduzi-las, mas vamos mais devagar. Seja G o 
baricentro. Ponha GE = x e BC = 2x. Do mesmo modo, GD = y e GB = 2y. Use 
Pitagoras nos triangulos BEG e GCD, some tudo e seja o que Deus quiser. Se nao 
der certo, mas vai dar, eh soh escrever. Amplexos, olavo.
Antonio Olavo da Silva Neto

 

Seja ABC um triângulo de lados BC, CA, AB cujas medidas sãorespectivamente 
iguais a a, b, c. Se D e E são os pontos médios de AC e ABrespectivamente, 
mostre que a mediana BD é perpendicular a CE se, e somentese, b² + c² = 5a² 
 
 
É suficente usar a fórmula que fornece a mediana?
 
Grato
 
 
 
 
 
_
Explore the seven wonders of the world
http://search.msn.com/results.aspx?q=7+wonders+worldmkt=en-USform=QBRE

[obm-l] combinatoria chico nery 69

2008-09-23 Por tôpico Antonio Neto

Quantos são os números de 5 algarismos que têm três de seus algarismos iguais e 
os outros algarismos diferentes entre si e diferente dos três algarismos 
iguais?   Bem, acho que é o seguinte: divida em casos.a) Os repetidos são 
iguais a zero, e, pelas condições, os outros não.Teremos uma possibilidade para 
escolher quem se repete, Bin(4, 3) para arrumá-los, pois o zero não pode 
começar, 9 escolhas para um dos lugares vagos e 8 para o outro. Multiplicando, 
são 288 casos.
 
b) Os repetidos são diferentes de zero e um dos outros é igual a zero.
Teremos 4 escolhas para colocar o zero, 9 escolhas para os repetidos, Bin(4, 3) 
para colocá-los e 8 escolhas para o último, pois só falta preencher uma vaga.
 
c) Nenhum dos algarismos é zero, o paraíso!
Temos 9 modos de escolher o repetido, Bin(5, 3) para colocá-lo, 8 modos de 
preencher um dos doi vagos e 7 modos de preencher o último, num total de 5040.
 
Somando tudo, achei 6480. Para conferir, refiz, usando o método ao qual me 
refiro para os alunos do ensino médio como total menos o que não presta, ou 
seja, ignorei a restrição de que o número não deve começar por zero e subtraí 
os inociados por zero. Como achei o mesmo resultado, ou está certo ou errei 
duas vezes.
[], olavo.
Antonio Olavo da Silva Neto
_
Invite your mail contacts to join your friends list with Windows Live Spaces. 
It's easy!
http://spaces.live.com/spacesapi.aspx?wx_action=createwx_url=/friends.aspxmkt=en-us

[obm-l] combinatoria chico nery 69

2008-09-23 Por tôpico Antonio Neto

Quantos são os números de 5 algarismos que têm três de seus algarismos iguais e 
os outros algarismos diferentes entre si e diferente dos três algarismos 
iguais?   Bem, acho que é o seguinte: divida em casos.a) Os repetidos são 
iguais a zero, e, pelas condições, os outros não.Teremos uma possibilidade para 
escolher quem se repete, Bin(4, 3) para arrumá-los, pois o zero não pode 
começar, 9 escolhas para um dos lugares vagos e 8 para o outro. Multiplicando, 
são 288 casos.
 
b) Os repetidos são diferentes de zero e um dos outros é igual a zero.
Teremos 4 escolhas para colocar o zero, 9 escolhas para os repetidos, Bin(4, 3) 
para colocá-los e 8 escolhas para o último, pois só falta preencher uma vaga.
 
c) Nenhum dos algarismos é zero, o paraíso!
Temos 9 modos de escolher o repetido, Bin(5, 3) para colocá-lo, 8 modos de 
preencher um dos doi vagos e 7 modos de preencher o último, num total de 5040.
 
Somando tudo, achei 6480. Para conferir, refiz, usando o método ao qual me 
refiro para os alunos do ensino médio como total menos o que não presta, ou 
seja, ignorei a restrição de que o número não deve começar por zero e subtraí 
os inociados por zero. Como achei o mesmo resultado, ou está certo ou errei 
duas vezes.
[], olavo.
Antonio Olavo da Silva Neto
_
News, entertainment and everything you care about at Live.com. Get it now!
http://www.live.com/getstarted.aspx

[obm-l] Ronaldinho canta.

2008-07-23 Por tôpico Antonio Neto

olha esse video Ronaldinho canta – Pense em mim. hehe 
_
News, entertainment and everything you care about at Live.com. Get it now!
http://www.live.com/getstarted.aspx

[obm-l] Eu recebi.

2007-11-20 Por tôpico Antonio Neto

Eu recebi, desculpe a demora na resposta, andei hospitalizado. Abracos, olavo.
Antonio Olavo da Silva Neto Date: Fri, 9 Nov 2007 00:38:35 -0200 From: [EMAIL 
PROTECTED] To: obm-l@mat.puc-rio.br Subject: [obm-l] Off topic...  
Desculpem a insistência com os testes,  Colegas tem recebido minhas 
mensagens, mas eu . não !!! Por isto  mais um teste para eu tentar 
descobrir o mistério...  Nehab 
= 
Instruções para entrar na lista, sair da lista e usar a lista em 
http://www.mat.puc-rio.br/~obmlistas/obm-l.html 
=
_
Connect to the next generation of MSN Messenger 
http://imagine-msn.com/messenger/launch80/default.aspx?locale=en-ussource=wlmailtagline

[obm-l] An�lise Combinat�ria

2007-10-21 Por tôpico Antonio Neto
Bem, como ninguém respondeu, aí vai: o que você quer é saber o número de soluções da equação x_1 + x_2 + x_3 + x_4 = 12, onde cada x_i é um inteiro não negativo. A resposta é Bin(15, 3) = 455, se não errei nada. A sugestão clássica é consultar o livro do Morgado, editado pelo IMPA. Para os mais velhinhos, como eu e alguns outros (não vou citar para não melindrá-los), o Prelúdio à Análise Combinatória, do Arago, Poppe e Raimundo. Abraços, olavo.
Antonio Olavo da Silva Neto


From: Gustavo Souza [EMAIL PROTECTED]Reply-To: obm-l@mat.puc-rio.brTo: obm-l@mat.puc-rio.brSubject: [obm-l] Análise CombinatóriaDate: Sun, 21 Oct 2007 01:32:12 -0300 (ART)
Estava tentando fazer esse exercicio de uma apostila e encontrei uma enorme dificuldade, se alguem pudesse me ajudar dando alguma explicação, lá vai:

(IBMEC) Um empresário precisa comprar um total de 12 automóveis para sua empresa. Os modelos selecionados foram: Honda Civic, Astra, Toyota Corolla e Santana. Sabendo-se que podem ser comprados de zero a 12 veículos de cada marca, de quantas maneiras o empresário poderá adquirir os 12 automóveis?


Obrigado.
Abra sua conta no Yahoo! Mail, o único sem limite de espaço para armazenamento! MSN Messenger: converse com os seus amigos online. Instale grátis. Clique aqui. 

=
Instruções para entrar na lista, sair da lista e usar a lista em
http://www.mat.puc-rio.br/~obmlistas/obm-l.html
=


Re: [obm-l] N�mero de divisores

2007-09-30 Por tôpico Antonio Neto
Voce multiplica por 2 se quiser incluir os divisores negativos, que sao os simetricos dos positivos. Abracos, olavo
Antonio Olavo da Silva Neto


From: "Henrique Rennó" [EMAIL PROTECTED]Reply-To: obm-l@mat.puc-rio.brTo: obm-l@mat.puc-rio.brSubject: Re: [obm-l] Número de divisoresDate: Fri, 28 Sep 2007 20:21:54 -0300On 9/28/07, Antonio Neto [EMAIL PROTECTED] wrote:  Certamente existe. Voce deve fatorar o numero, somar uma unidade a cada  expoente obtido e multiplicar esses numeros. Pensem no motivo de somar uma  unidade a cada. Isso dah o numero de divisores positivos, para o total  multiplique por 2. Abracos, olavo.O motivo de adicionar a unidade é porque o expoente pode ser 0. Masnão entendi por que multiplicar por 2?--Henrique=Instruções para entrar na lista, 
sair da lista e usar a lista emhttp://www.mat.puc-rio.br/~nicolau/olimp/obm-l.html=MSN Messenger: converse com os seus amigos online. Instale grátis. Clique aqui. 

=
Instruções para entrar na lista, sair da lista e usar a lista em
http://www.mat.puc-rio.br/~nicolau/olimp/obm-l.html
=


Re: [obm-l] N�mero de divisores

2007-09-28 Por tôpico Antonio Neto
 Certamente existe. Voce deve fatorar o numero, somar uma unidade a cada expoente obtido e multiplicar esses numeros. Pensem no motivo de somar uma unidade a cada. Isso dah o numero de divisores positivos, para o total multiplique por 2. Abracos, olavo.
Antonio Olavo da Silva Neto


From: ralonso [EMAIL PROTECTED]Reply-To: obm-l@mat.puc-rio.brTo: obm-l@mat.puc-rio.brSubject: Re: [obm-l] Número de divisoresDate: Thu, 27 Sep 2007 14:20:15 -0300Deve ser algo que envolva combinatória de primos da fatoração do número ou soma de números obtidos por análise combinatória. Pense por exemplo no número fatorado: 2^3 * 5 * 7^2
As combinações (divisores) são: 2 2^2 2^3 5 7 7^2
2*5 2^2 * 5 ...
Não sei se existe uma fórmula fechada, mas creio que deva existir. Abraço.
Ronaldo.
[EMAIL PROTECTED] wrote:


 Oi gente! Entrei na lista recentemente e queria saber, Existe alguma fórmula para calcular o número de divisores de um número? De 2004, por exemplo.. Agradeço desde já. Abraços.MSN Messenger: converse com os seus amigos online. Instale grátis. Clique aqui. 

=
Instruções para entrar na lista, sair da lista e usar a lista em
http://www.mat.puc-rio.br/~nicolau/olimp/obm-l.html
=


[obm-l] Contagem - fun�

2007-09-27 Por tôpico Antonio Neto
 Oi, Bruna,
Uma funcao consiste em atribuir uma unica imagem para cada elemento do dominio. Estou supondo que voce estah familiarizada com o principio de multiplicacao. Entao, para o primeiro elemento do dominio, temos n escolhas, os n elementos do conjunto B. o mesmo para o segundo, terceiro, e assim por diante, ateh o ultimo, o que dah m elevado a n. Se ainda nao estiver claro, escreva de novo, sou um menino da geracao do Nehab. Abracos, olavo.
Antonio Olavo da Silva Neto


From: "Bruna Carvalho" [EMAIL PROTECTED]Reply-To: obm-l@mat.puc-rio.brTo: obm-l@mat.puc-rio.brSubject: Re: [obm-l] Contagem - funçãoDate: Wed, 26 Sep 2007 11:31:53 -0300Olá Antonio Neto, eu agradeceria por mais informações sobre esse assunto. MSN Messenger: converse com os seus amigos online. Instale grátis. Clique aqui. 

=
Instruções para entrar na lista, sair da lista e usar a lista em
http://www.mat.puc-rio.br/~nicolau/olimp/obm-l.html
=


Re: [obm-l] Contagem - fun�

2007-09-27 Por tôpico Antonio Neto
 Oi de novo, Bruna,
 veja que se trata de escolher as imagens dos elementos do dominio, receio que a minha didatica esteja falhando, para cada elemento voce tem n escolhas, pois cada elemento de B pode ser imagem, o que vai dar m.m.m...m, ou seja, m elevado a n. Qualquer coisa, escreva. Abracos, olavo.
Antonio Olavo da Silva Neto


From: "Bruna Carvalho" [EMAIL PROTECTED]Reply-To: obm-l@mat.puc-rio.brTo: obm-l@mat.puc-rio.brSubject: Re: [obm-l] Contagem - funçãoDate: Wed, 26 Sep 2007 16:47:46 -0300Olá meninos.Então, só não fico claro pra mim pq aplicar o Princípio Multiplicativo.obrigada pela atenção.bjosMSN Messenger: converse com os seus amigos online. Instale grátis. Clique aqui. 

=
Instruções para entrar na lista, sair da lista e usar a lista em
http://www.mat.puc-rio.br/~nicolau/olimp/obm-l.html
=


Re: [obm-l] Fun��o

2007-09-25 Por tôpico Antonio Neto
Acredito que 6, afinal as funções são CRESCENTES, e não NÃO DECRESCENTES. Abraços, olavo.
Antonio Olavo da Silva Neto


From: "Bruna Carvalho" [EMAIL PROTECTED]Reply-To: obm-l@mat.puc-rio.brTo: obm-l@mat.puc-rio.brSubject: Re: [obm-l] FunçãoDate: Mon, 24 Sep 2007 17:30:17 -0300olá meninos.agora tenho uma dúvida, são 10 ou 6 possibilidades de funcoes crescentes ?eu não tenho gabarito.bjos meninos. MSN Messenger: converse com os seus amigos online. Instale grátis. Clique aqui. 

=
Instruções para entrar na lista, sair da lista e usar a lista em
http://www.mat.puc-rio.br/~nicolau/olimp/obm-l.html
=


[obm-l] Contagem - fun�

2007-09-25 Por tôpico Antonio Neto
n elevado a m. A ideia eh que para cada elemento de A hah n escolhas para a imagem, logo voce tem n.n. ... n escolhas. Se quiser mais informacoes, escreve. Abraco, olavo.
Antonio Olavo da Silva Neto


From: "Bruna Carvalho" [EMAIL PROTECTED]Reply-To: obm-l@mat.puc-rio.brTo: obm-l@mat.puc-rio.brSubject: [obm-l] Contagem - funçãoDate: Tue, 25 Sep 2007 14:45:21 -0300A e B são conjuntos tais que #A=m e #B=n. Quantas funções de A em B existem?-- Bjos, Bruna MSN Messenger: converse com os seus amigos online. Instale grátis. Clique aqui. 

=
Instruções para entrar na lista, sair da lista e usar a lista em
http://www.mat.puc-rio.br/~nicolau/olimp/obm-l.html
=


[obm-l] 17 professores

2007-09-23 Por tôpico Antonio Neto
Lema: Vamos comecar com seis professores que se correspondem sobre dois assuntos, Fisica e Matematica. Provaremos que entre eles hah 3 que se correspondem sobre o mesmo assunto. Representando os professores por pontos, Matematica por azul e Fisica por vermelho, ligaremos dois ponto em azul quando os dois professores associados a esses pontos se correspondem sobre Matematica, e o mesmo para Fisica, em vermelho. O que queremos provar eh que existe um triangulo cromatico (isto eh, com os tres lados da mesma cor). 
Considere o professor A. No grafico, saem dele 5 segmentos. Logo, como soh hah duas cores, no minimo 3 sao da mesma cor, digamos vermelho. Se algum dos segmentos ligando esses tres pontos eh vermelho, a afirmativa estah demonstrada. Caso contrario, todos sao azuis, e provamos a afirmativa.
Agora vamos ao famigerado caso dos 17. Como anteriormente, seja Matemativa azul e Fisica vermelho. Introduzimos Quimica preto. Considere o professor A. No diagrama saem dele 16 segmentos, vermelhos azuis ou pretos. Como soh hah 3 cores, pelo menos 6 devem ter a mesma cor, digamos preto. Se algum dos segmentos ligando esses 6 pontos for preto, provamos o famigerado. Caso contrario, serao todos azuis ou vermelhos, e caimos no lema acima.
Perguntinha: Qual o numero minimo de professores, correspondendo-se sobre quatro assuntos, para formar um triangulo cromatico? Abracos, olavo.
Antonio Olavo da Silva Neto


From: Rogerio Ponce [EMAIL PROTECTED]Reply-To: obm-l@mat.puc-rio.brTo: obm-l@mat.puc-rio.brSubject: [obm-l] 17 professoresDate: Mon, 17 Sep 2007 15:28:13 -0300 (ART)Ola' pessoal,numa escola, ha' um grupo de 17 professores que se correspondem de tal forma que quaisquer 2 professores deste grupo trocam ideias sobre exatamente um assunto fixo entre matematica, fisica ou quimica.Prove que ha' pelo menos 3 professores que se correspondem sobre o mesmo assunto (isto e', a correspondencia entre A e B, B e C, assim como entre A e C sao sobre o mesmo assunto).[]'sRogerio PoncePS: nao confundir com "prove que ha' 3 professores que enviam alguma correspondencia sobre um mesmo assunto", que se resolve trivialmente com o "principipo da casa de pombos" (e ja' seria verdadeira para um grupo de 5 professores).
Flickr agora em português. Você clica, todo mundo vê. Saiba mais. MSN Messenger: converse com os seus amigos online. Instale grátis. Clique aqui. 

=
Instruções para entrar na lista, sair da lista e usar a lista em
http://www.mat.puc-rio.br/~nicolau/olimp/obm-l.html
=


[obm-l] ESTUDANTE

2007-09-12 Por tôpico Antonio Neto
Receio que nao seja bem assim. Se no primeiro grupo ele escolhe, por exemplo, Analise e Topologia, eh o mesmo que escolher Topologia e Analise, isto eh, o numero de escolhas eh Bin(4, 2) = 6, e o total eh 6x6 = 36. Abracos, olavo.
Antonio Olavo da Silva Neto


From: "Marcelo Salhab Brogliato" [EMAIL PROTECTED]Reply-To: obm-l@mat.puc-rio.brTo: obm-l@mat.puc-rio.brSubject: Re: [obm-l] ESTUDANTEDate: Wed, 12 Sep 2007 16:42:14 -0300Olá Arkon,ele pode escolher as 2 do 1o. grupo de 4*3 = 12 maneiras diferentes..o mesmo vale para o segundo grupo..assim, ele pode escolher em 12*12 = 144 maneiras diferentesabraços,SalhabOn 9/12/07, arkon [EMAIL PROTECTED] wrote: Alguém pode, por favor, resolver esta: (UFPB-85) Um estudante, ao ingressar numa escola, deve cursar 4 disciplinas  de sua escolha, sendo 2 delas num dado grupo de 4 disciplinas e as outras 2  noutro grupo de 4 
disciplinas. De quantas maneiras ele pode fazer a sua  opção? DESDE JÁ MUITO OBRIGADO=Instruções para entrar na lista, sair da lista e usar a lista emhttp://www.mat.puc-rio.br/~nicolau/olimp/obm-l.html=MSN Messenger: converse com os seus amigos online. Instale grátis. Clique aqui. 

=
Instruções para entrar na lista, sair da lista e usar a lista em
http://www.mat.puc-rio.br/~nicolau/olimp/obm-l.html
=


RE: [obm-l] UFPB-77

2007-09-06 Por tôpico Antonio Neto
Eh verdade, esta eh a equacao do eixo radical das duas circunferencias, a reta cujos pontos tem a mesma potencia em relacao aos dois circulos. Inclusive os raios nao precisam ser iguais, basta "tirar os parenteses" e subtrair as duas equacoes. Abracos, olavo.
Antonio Olavo da Silva Neto


From: Anselmo Alves de Sousa [EMAIL PROTECTED]Reply-To: obm-l@mat.puc-rio.brTo: obm-l@mat.puc-rio.brSubject: RE: [obm-l] UFPB-77Date: Thu, 6 Sep 2007 20:08:47 +0300


Considere o círculo com centro em (a,b). Temos (x-a)^2+(x-b)^2=R^2tomemos outro círculo com centro em (c,d). Sua equação serah (x-c)^2+(y-d)^2=R^2Tomando a igualdade, teremos:(x-a)^2+(x-b)^2=(x-c)^2+(y-d)^2daí [(x-a)^2-(x-c)^2]+[(y-b)^2-(y-d)^2]=0[(x-a)-(x-c)][(x-a)+(x-c)]+[(y-b)-(y-d)][(y-b)+(y-d)]=0[(-a-c)][(2x-a-c)]+[(-b-d)][(2y-b-d)]=0que nos dah(2y-b-d)= - [(-a-c)][(2x-a-c)]2y=- {[(-a-c)][(2x-a-c)]/(-b-d)}+b+dque eh a equação de uma reta."O muito estudar eh enfado para a carne" (Rei Salomão)


Date: Thu, 6 Sep 2007 13:27:02 -0300Subject: [obm-l] UFPB-77From: [EMAIL PROTECTED]To: obm-l@mat.puc-rio.br


Pessoal alguém, por favor, pode responder esta

(UFPB-77) A união de todos os círculos de raio R, num mesmo plano, passando por um ponto fixo é:

a) um ponto. b) duas retas. c) uma reta. d) um círculo. e) o plano.DESDE JÁ MUITO OBRIGADO

Encontre o que procura com mais eficiência! Instale já a Barra de Ferramentas com Windows Desktop Search GRÁTIS! Experimente já! MSN Messenger: converse com os seus amigos online. Instale grátis. Clique aqui. 

=
Instruções para entrar na lista, sair da lista e usar a lista em
http://www.mat.puc-rio.br/~nicolau/olimp/obm-l.html
=


[obm-l] TELEFONE.1

2007-09-04 Por tôpico Antonio Neto
Receio que não seja bem isso, o prexo foi esquecido (ou, na pressa, nao 
lido). Para o prefixo, soh hah uma possibilidade. Para os dois ultimos soh 3 
possibilidades, pois (3, 6) nao pode ser utilizado, o 6 jah estah no 
prefixo, e os algarismos sao distintos. Sobram dois espacos para preencher, 
e voce dispoe de 5 algarismos, logo sao 5x4 = 20 possibilidades. Se nao 
errei nada, sao 3x20 = 60 telefones.




From: saulo nilson [EMAIL PROTECTED]
Reply-To: obm-l@mat.puc-rio.br
To: obm-l@mat.puc-rio.br
Subject: Re: [obm-l] TELEFONE.1
Date: Tue, 4 Sep 2007 14:30:01 -0300

0123456789 sao os numeros que vc coloca no telefone, mas so
1,2
2,4
3,6
4,8
so os numeros possiveis para os ultimo e o penultimo
entao temos
numero de maneiras de preencher os ultimos 2 digitos, 4 depois disso sobram
8 numeros para preencher o restante, logo sao
4*8!/2!=8!

On 9/3/07, arkon [EMAIL PROTECTED] wrote:

  Olá pessoal, alguém pode resolver, por favor, esta

  Quantos números de telefones com prefixo 567 existem no Guará, com 
todos
 os algarismos distintos e o último algarismo igual ao dobro do 
penúltimo?

 Considere que os telefones têm números com 7 algarismos.

 DESDE JÁ MUITO OBRIGADO



_
MSN Messenger: instale grátis e converse com seus amigos. 
http://messenger.msn.com.br


=
Instruções para entrar na lista, sair da lista e usar a lista em
http://www.mat.puc-rio.br/~nicolau/olimp/obm-l.html
=


[obm-l] Um numero N com n algarismos....

2007-08-01 Por tôpico Antonio Neto
Experimente também os inversos de 3, 11, 31, 37, 41, 43, 53, 67, 71, 73, 79, 
83 e 89, para ficarmos nos menores que 100. A explicação é via congruência 
módulo p, onde p é o primo. Boas contas, abraço, olavo.




From: Bruno França dos Reis [EMAIL PROTECTED]
Reply-To: obm-l@mat.puc-rio.br
To: obm-l@mat.puc-rio.br
Subject: Re: [obm-l] Um numero N com n algarismos
Date: Wed, 1 Aug 2007 20:38:33 +0200

Acho interessante essas propriedades dos inversos desses primos que vc
citou.

Uma coisa que fico um tanto quanto intrigado também é o caso do período do
inverso do 13. Vc pode quebrar ao meio esse período. Pegando o número
menor, se vc for multiplicando por 2, 3, 4, ..., assim como fez com o
período do 1/7, vc vai obtendo também permutações cíclicas de um dos dois
períodos, eles vão se alternando.

Abraço
Bruno


2007/8/1, Antonio Neto [EMAIL PROTECTED]:


   Para n algarismos, a solução que me ocorre é a mesma de todos os que 
já

 responderam. Mas se o n é dado, há soluções mais diretas, como esta, do
 Colégio Naval, se não me falha a velhaca:
Um número de seis algarismos começa à esquerda pelo algarismo 1.
 Retirando o 1 inicial e colocando-o à direita do número, o novo número
 obtido é o triplo do original.
   Se chamarmos o número de 5 algarismos obtido pela supressão do 1 de x, 
é
 só fazer 3(10 + x) = 10x + 1, e o número original é 142857, que 
aliás

 é
 o período de 1/7. Experimentem multiplicar 142857 por 2, 3, 4, 5, 6 e 7.
 Depois por (oh, surpresa!!!) 8, 9, ... Números com esta propriedade são
 chamados de números cíclicos. Os primeiros são os períodos dos inversos 
de

 7, 17, 19, 23, 29, 47, 59, 61 e 97. A minha fonte é o livro do Albert H.
 Beiler, Recreations in the Theory of Numbers, da Dover, mas deve haver
 muito na internet, estou respondendo meio às pressas. Abraços, olavo.


_
MSN Messenger: instale grátis e converse com seus amigos. 
http://messenger.msn.com.br


=
Instruções para entrar na lista, sair da lista e usar a lista em
http://www.mat.puc-rio.br/~nicolau/olimp/obm-l.html
=


[obm-l] Um numero N com n algarismos....

2007-07-31 Por tôpico Antonio Neto


  Para n algarismos, a solução que me ocorre é a mesma de todos os que já 
responderam. Mas se o n é dado, há soluções mais diretas, como esta, do 
Colégio Naval, se não me falha a velhaca:
   Um número de seis algarismos começa à esquerda pelo algarismo 1. 
Retirando o 1 inicial e colocando-o à direita do número, o novo número 
obtido é o triplo do original.
  Se chamarmos o número de 5 algarismos obtido pela supressão do 1 de x, é 
só fazer 3(10 + x) = 10x + 1, e o número original é 142857, que aliás é 
o período de 1/7. Experimentem multiplicar 142857 por 2, 3, 4, 5, 6 e 7. 
Depois por (oh, surpresa!!!) 8, 9, ... Números com esta propriedade são 
chamados de números cíclicos. Os primeiros são os períodos dos inversos de 
7, 17, 19, 23, 29, 47, 59, 61 e 97. A minha fonte é o livro do Albert H. 
Beiler, Recreations in the Theory of Numbers, da Dover, mas deve haver 
muito na internet, estou respondendo meio às pressas. Abraços, olavo.





From: vitoriogauss [EMAIL PROTECTED]
Reply-To: obm-l@mat.puc-rio.br
To: obm-l obm-l@mat.puc-rio.br
Subject: [obm-l] Um numero N com n algarismos
Date: Tue, 31 Jul 2007 15:01:58 -0300

Ola' pessoal,

Uma ajuda Considere um número N com n algarismos e na posição das 
unidades o número 2. Ao invertemos o 2, colocando-o na posição inicial, 
encontramos um novo número K, onde K=2N. Qual o valor de N?


Pensei em congruencia...seria uma boa


=
Instruções para entrar na lista, sair da lista e usar a lista em
http://www.mat.puc-rio.br/~nicolau/olimp/obm-l.html
=


_
MSN Messenger: instale grátis e converse com seus amigos. 
http://messenger.msn.com.br


=
Instruções para entrar na lista, sair da lista e usar a lista em
http://www.mat.puc-rio.br/~nicolau/olimp/obm-l.html
=


[obm-l] Um numero N com n algarismos....

2007-07-31 Por tôpico Antonio Neto


  Olhem o período de 1/19. Abraços, olavo.



From: vitoriogauss [EMAIL PROTECTED]
Reply-To: obm-l@mat.puc-rio.br
To: obm-l obm-l@mat.puc-rio.br
Subject: Re: [obm-l] Um numero N com n algarismos
Date: Tue, 31 Jul 2007 20:02:44 -0300

um colega que me deu
vou começar agora a pensar nela

Oi, Vitorio,

 Semelhante a esta (acho que foi a original...) caiu na Olimpíada de
 Maio de 2001:
 A solução é armar a conta e fazê-la, mesmo

 Sara escreveu no quadro negro um número inteiro de menos de 30
 algarismos e que termina em 2.  Célia apaga o 2 do fim e escreve-o no
 início. O número que fica é igual ao dobro do número que tinha
 escrito Sara. Qual o número que Sara escreveu?

 Solução
 Do enunciado, temos:
 ?  g   f   e  d   c   b  a  2
 x  2
 2 ...  h   g   f  e  d   c   b  a

 a = 4; colocando o 4 no lugar do a na parcela de cima  e continuando
 a multiplicação, obtemos, b = 8 (2 x 4);  assim, continuando o mesmo
 mecanismo, temos, sucessivamente, c = 6 (2 x 8 = 16, e vai um); d =
 3; e = 7; f = 4...
 Continuando o processo até que ocorra o algarismo 2 pela primeira
 vez, obtemos o número desejado:
 210.526.315.789.473.684

 Observe que se não limitarmos o número de algarismos, haverá outras
 soluções (é só continuar a brincadeira).

 Abraços,
 Nehab

 PS: Onde você viu esta questão?


 At 15:01 31/7/2007, you wrote:
 Ola' pessoal,
 
 Uma ajuda Considere um número N com n algarismos e na posição
 das unidades o número 2. Ao invertemos o 2, colocando-o na posição
 inicial, encontramos um novo número K, onde K=2N. Qual o valor de N?
 
 Pensei em congruencia...seria uma boa
 
 
 
=

 Instruções para entrar na lista, sair da lista e usar a lista em
 http://www.mat.puc-rio.br/~nicolau/olimp/obm-l.html
 
=



Vitório Gauss


=
Instruções para entrar na lista, sair da lista e usar a lista em
http://www.mat.puc-rio.br/~nicolau/olimp/obm-l.html
=


_
MSN Messenger: instale grátis e converse com seus amigos. 
http://messenger.msn.com.br


=
Instruções para entrar na lista, sair da lista e usar a lista em
http://www.mat.puc-rio.br/~nicolau/olimp/obm-l.html
=


[obm-l] Mais um de an�lise

2007-07-27 Por tôpico Antonio Neto


  Receio que não seja isso. Suponha um grupo de 4 cientistas em frente ao 
armario. Pelo enunciado, eles não conseguirao abri-lo. Logo, existe pelo 
menos um cadeado do qual eles nao tem a chave. Se trocarmos esse grupo de 4 
por qualquer outro, ocorrerá o mesmo. Assim, o número de cadeados será igual 
no mínimo ao número de grupos de 4 cientistas. Esse numero eh Bin(9, 4) = 
126.
  Vejamos agora as chaves. Se houver um grupo de 4 cientistas em frente ao 
armario, o quinto que chegar deve ter a chave que eles nao tem. Assim, cada 
cientista deve ser capaz de abrir o cadeado que falta para qualquer grupo de 
4 formado pelos outros 8. Este numero eh Bin(8, 4) = 70. Abracos, olavo.




From: Marcelo Salhab Brogliato [EMAIL PROTECTED]
Reply-To: obm-l@mat.puc-rio.br
To: obm-l@mat.puc-rio.br
Subject: Re: [obm-l] Mais um de análise
Date: Wed, 25 Jul 2007 22:57:55 -0300

Olá,

apenas uma curiosidade.. podemos pensar em um polinomio de grau 5 e
dizer que a chave é f(x0).. falamos um ponto (x, f(x)) para cada
cientista.. qdo 5 ou mais estiverem presente é possível abrir o
cadeado.. pois atraves de interpolacao obtem-se f(x0).. alguem ve
problemas nesse metodo?


queremos que 4 nao abram o cadeado ao mesmo tempo..
isto é.. 4 juntos tem q faltar pelo menos 1 chave..
digamos que falte exatamente 1 chave.. entao os outro 5 tem que ter essa 
chave..

partindo dessa ideia, vamos supor que temos 5 copias das chaves de
cada cadeado..

partindo da ideia de que cada cientista tem o mesmo numero de chaves,
temos: 5n = 9k
n = numero de cadeados
k = numero de chaves com cada cientista

hmm nao sei explicar como, mas tive a seguinte ideia..
pegue as 5 chaves do cadeado 1... de para os cientistas 1,2,3,4,5...
agora pegue as 5 chaves do cadeado 2... de para os cientistas 2,3,4,5,6...
faca o mesmo para os demais cadeados.. qdo chegar em 9, volte para 1..

matematicamente, vamos enumerar os cientistas de 0 à 8.. e os cadeados
de 0 à n-1
as chaves do cadeado k serao dadas ao cientistas k, k+1, k+2, k+3,
k+4... todos modulo 9..

vamos usar a seguinte notacao: cadeado k: cientistas com chave deste 
cadeado

cadeado 0: 0, 1, 2, 3, 4
cadeado 1: 1, 2, 3, 4, 5
cadeado 2: 2, 3, 4, 5, 6
cadeado 3: 3, 4, 5, 6, 7
cadeado 4: 4, 5, 6, 7, 8

neste ponto, vemos que o cientista 4 tem 5 chaves.. logo, vamos deixar
todos assim..
cadeado 5: 5, 6, 7, 8, 0
cadeado 6: 6, 7, 8, 0, 1
cadeado 7: 7, 8, 0, 1, 2
cadeado 8: 8, 0, 1, 2, 3

assim, com 9 cadeados.. 5 copias de cada chave.. conseguimos que
apenas 5 consigam acessar o segredo..

mass... nao sei como provar que esse eh o numero minimo de cadeados..
usando minhas hipoteses, temos que: 5n = 9k ... n=9 e k=5 sao os
menores inteiros que satisfazem a relacao.. mas parti de 2 hipoteses:
mesmo numero de chave com cada cientista e qdo temos apenas 4
cientistas, falta apenas 1 chave...

da pra generalizar minha ideia pra c cientistas e pra abrir com no
minimo m..
abracos,
Salhab




On 7/25/07, MauZ [EMAIL PROTECTED] wrote:
 Olá

 esse gostaria que me ajudassem, parece mto interessante:

 Nove cientistas trabalham num projeto sigiloso. Por questões de 
segurança,
 os planos são guardados num cofre protegido por muitos cadeados de modo 
que
 só é possível abri-los todos se houver pelo menos 5 cientistas 
presentes.

 a) Qual é o numero mínimo possível de cadeados?
 b) Na situação do item a, quantas chaves cada um deve ter?


 Agradeço a quem fizer e da mesma forma a quem tentar,

 Maurizio


=
Instruções para entrar na lista, sair da lista e usar a lista em
http://www.mat.puc-rio.br/~nicolau/olimp/obm-l.html
=


_
MSN Messenger: instale grátis e converse com seus amigos. 
http://messenger.msn.com.br


=
Instruções para entrar na lista, sair da lista e usar a lista em
http://www.mat.puc-rio.br/~nicolau/olimp/obm-l.html
=


[obm-l] polin�mios

2007-07-17 Por tôpico Antonio Neto


  Amigos, sugiro uma olhada no livro Curso de Algebra, do Sinezio de 
Farias. A edicao que tenho eh de 1959 (sou da turma do Nehab), e as 
definicoes sao meio diferentes do que aprendi. Ele admite monomios da forma 
(ax^n.y^m)/(bz^p.t^q), onde a e b são constantes e m, n, p e q sao numeros 
naturais. Quando me ensinaram, o que foi dito é que monomio eh ax^m.y^n.z^p, 
onde a eh constante e m, n e p inteiros positivos. A impressao que tenho eh 
que tudo eh convencao (por favor, nao aquela beberagem), nao vejo utilidade 
na definicao do Sinezio. Soh para ter ideia, ele chama inteiros relativos de 
numeros qualificados. Eh bom lembrar que a primeira edicao eh de 1946. 
Outro exemplo: antigamente matrizes e determinantes tinham (quem lembra?) 
diagonal principal e diagonal secundaria. Hoje, qualquer um com uma leve 
camada de verniz de civilizacao nao usa mais essa bobeira, exceto muitos 
professores que todos nos conhecemos. Acho que eh por ahi. Abracos, olavo




From: Antonio Giansante [EMAIL PROTECTED]
Reply-To: obm-l@mat.puc-rio.br
To: obm-l@mat.puc-rio.br
Subject: Re: [obm-l] polinômios
Date: Mon, 16 Jul 2007 22:16:26 -0300 (ART)

Pois então jones, mas mesmo qdo se fala em anel, só
encontrei definições com coeficientes inteiros. O
problema é que, em alguns casos, falamos de polinômios
com qualquer grau real (como no bin. de Newton),
entretanto, não se define o grau do polinômio nesses
casos. Eu não encontrei nenhuma referência a esses
casos, e não quero criar novas teorias só pra
fingir que respondi aos meus alunos. Quero passar a
informação correta. Em vista disso, podes me indicar
alugma referência bibliográfica? obrigado.


--- jones colombo [EMAIL PROTECTED] escreveu:

  Olha, não sei muito bem,  mas esta é uma questão de
 definição de
 polinômio.  Falamos que um  elemento  é um polinômio
 quando é formado por
 combinações linear de monomios, e os monomios
 aparecem com coeficientes
 inteiros positivos. Nada o impede de trabalhar com
 outras variantes deste
 objeto, mas então é costume falar que o objeto é um
 anel com tais e tais
 propriedades. Creio que esta é a convenção.

  Até.
 Jones

 On 7/16/07, Antonio Giansante
 [EMAIL PROTECTED] wrote:
 
  olá. Alguém chegou a alguma conclusão com relação
 à
  minha pergunta? Qualquer pista já me ajuda.
 Valeu.
 
 
 
 
 
 


  Novo Yahoo! Cadê? - Experimente uma nova busca.
  http://yahoo.com.br/oqueeuganhocomisso
 

=
  Instruções para entrar na lista, sair da lista e
 usar a lista em
 
 http://www.mat.puc-rio.br/~nicolau/olimp/obm-l.html
 

=
 






Novo Yahoo! Cadê? - Experimente uma nova busca.
http://yahoo.com.br/oqueeuganhocomisso
=
Instruções para entrar na lista, sair da lista e usar a lista em
http://www.mat.puc-rio.br/~nicolau/olimp/obm-l.html
=


_
MSN Messenger: instale grátis e converse com seus amigos. 
http://messenger.msn.com.br


=
Instruções para entrar na lista, sair da lista e usar a lista em
http://www.mat.puc-rio.br/~nicolau/olimp/obm-l.html
=


[obm-l] polin�mios

2007-07-17 Por tôpico Antonio Neto


  Amigos, sugiro uma olhada no livro Curso de Algebra, do Sinezio de 
Farias. A edicao que tenho eh de 1959 (sou da turma do Nehab), e as 
definicoes sao meio diferentes do que aprendi. Ele admite monomios da forma 
(ax^n.y^m)/(bz^p.t^q), onde a e b são constantes e m, n, p e q sao numeros 
naturais. Quando me ensinaram, o que foi dito é que monomio eh ax^m.y^n.z^p, 
onde a eh constante e m, n e p inteiros positivos. A impressao que tenho eh 
que tudo eh convencao (por favor, nao aquela beberagem), nao vejo utilidade 
na definicao do Sinezio. Soh para ter ideia, ele chama inteiros relativos de 
numeros qualificados. Eh bom lembrar que a primeira edicao eh de 1946. 
Outro exemplo: antigamente matrizes e determinantes tinham (quem lembra?) 
diagonal principal e diagonal secundaria. Hoje, qualquer um com uma leve 
camada de verniz de civilizacao nao usa mais essa bobeira, exceto muitos 
professores que todos nos conhecemos. Acho que eh por ahi. Abracos, olavo




From: Antonio Giansante [EMAIL PROTECTED]
Reply-To: obm-l@mat.puc-rio.br
To: obm-l@mat.puc-rio.br
Subject: Re: [obm-l] polinômios
Date: Mon, 16 Jul 2007 22:16:26 -0300 (ART)

Pois então jones, mas mesmo qdo se fala em anel, só
encontrei definições com coeficientes inteiros. O
problema é que, em alguns casos, falamos de polinômios
com qualquer grau real (como no bin. de Newton),
entretanto, não se define o grau do polinômio nesses
casos. Eu não encontrei nenhuma referência a esses
casos, e não quero criar novas teorias só pra
fingir que respondi aos meus alunos. Quero passar a
informação correta. Em vista disso, podes me indicar
alugma referência bibliográfica? obrigado.


--- jones colombo [EMAIL PROTECTED] escreveu:

  Olha, não sei muito bem,  mas esta é uma questão de
 definição de
 polinômio.  Falamos que um  elemento  é um polinômio
 quando é formado por
 combinações linear de monomios, e os monomios
 aparecem com coeficientes
 inteiros positivos. Nada o impede de trabalhar com
 outras variantes deste
 objeto, mas então é costume falar que o objeto é um
 anel com tais e tais
 propriedades. Creio que esta é a convenção.

  Até.
 Jones

 On 7/16/07, Antonio Giansante
 [EMAIL PROTECTED] wrote:
 
  olá. Alguém chegou a alguma conclusão com relação
 à
  minha pergunta? Qualquer pista já me ajuda.
 Valeu.
 
 
 
 
 
 


  Novo Yahoo! Cadê? - Experimente uma nova busca.
  http://yahoo.com.br/oqueeuganhocomisso
 

=
  Instruções para entrar na lista, sair da lista e
 usar a lista em
 
 http://www.mat.puc-rio.br/~nicolau/olimp/obm-l.html
 

=
 






Novo Yahoo! Cadê? - Experimente uma nova busca.
http://yahoo.com.br/oqueeuganhocomisso
=
Instruções para entrar na lista, sair da lista e usar a lista em
http://www.mat.puc-rio.br/~nicolau/olimp/obm-l.html
=


_
MSN Messenger: instale grátis e converse com seus amigos. 
http://messenger.msn.com.br


=
Instruções para entrar na lista, sair da lista e usar a lista em
http://www.mat.puc-rio.br/~nicolau/olimp/obm-l.html
=


Re: [obm-l] Estatistica

2007-07-15 Por tôpico Antonio Neto


  Voces estao certos. MUITO excepcionalmente, este gabarito esta errado no 
livro. Na ultima edicao ele ja foi consertado. Abracos, olavo.




From: Graciliano Antonio Damazo [EMAIL PROTECTED]
Reply-To: obm-l@mat.puc-rio.br
To: obm-l@mat.puc-rio.br
Subject: Re: [obm-l] Estatistica
Date: Sun, 15 Jul 2007 18:01:44 -0300 (ART)

Olá Marcelo SAlhad mais uma vez por esclarecer minhas duvidas, quanto a 
resposta que tenho dessa segunda questao é o dobro dessa que voce chegou, 
que foi a mesma que eu cheguei... esse exercicio esta no livro Analise 
Combinatoria e Probabilidade da OBM..


  obrigado e abraços...

  graciliano

Marcelo Salhab Brogliato [EMAIL PROTECTED] escreveu:
  Olá,

2) vou tentar a segunda agora..
primeiro vamos pegar 5 dezenas de 1 linha.. C(10, 5)
agora, vamos pegar 1 dezena de outra linha.. C(10, 1)
legal.. para a primeira escolha, temos 5 linhas.. logo: 5 * C(10, 5)
agora, para a segunda escolha, temos 4 linhas.. logo: 4 * C(10, 1)
portanto, os casos favoraveis sao: 5 * C(10, 5) * 4 * C(10, 1)

vamos ver todos os casos possiveis..
hmm.. C(50, 6)

P = 20 * C(10, 5) * C(10, 1) / C(50, 6) = 20 * 252 * 10 / 15890700 = 
0,003171

P = 0,3171%
é isso?

abracos,
Salhab



On 7/13/07, Graciliano Antonio Damazo wrote:
 Ai vao duas questoes em que encontrei dificuldade:

 a primeira a minha dificuldade foi em que o numero de bolas sorteadas 
eram
 diferentes do numero de bolas escolhidas, ai eu me perdi em montar os 
casos

 possiveis e favoraveis e acho que resolvi errado

 1) Na Sena sao sorteados 5 dezenas entre 01 - 02 - ... - 99 - 00, e o
 apostador pode escolher 10 dezenas. Qual a probabilidade do apostador
 acertar a terna( 3 dezenas)?

 na segunda questao, eu tenho a resposta e a minha resolução dá 
exatamente o

 dobro da resposta.

 2) Supondo que na Loto as dezenas 01 - 02 - ... - 50 nas cartelas sao
 dispostas em 5 linhas e 10 colunas. Sabendo que sao sorteadas 6 dezenas,
 qual a probabilidade dessas dezenas ocuparem exatemente duas linhas, com 
5

 dezenas em uma e 1 dezena em outra?

 A minha soluçõa foi:

 OBS: represento por C(n,p) a combinação de n elementos para escolher p, 
e

 Pn, por uma permutação de n elemntos.

 casos favoraveis: primeiro temos que escolher 2 linhas dentre as 5, o 
que
 pode ser feito de C(5,2) maneiras. Ainda temos que escolher 5 dezenas em 
uma
 linha e 1 dezenas na outra linha que pode ser feito de C(10,5)*C(10,1). 
Além
 disso, podemos escolher 5 na primeira linha e 1 na segunda linha 
escolhida,

 ou vice versa, entao temos que multiplicar o resultado por P2.

 casos possiveis: C(50,6)

 portanto eu encontrei como resposta essa expressao:

 probabilidade = C(5,2)*C(10,5)*C(10,1)*P2 / C(50,6)

 gostaria da ajuda de voces mais uma vez galeraagradeço desde já

 abraços

 Graciliano

 
 Novo Yahoo! Cadê? - Experimente uma nova busca.



=
Instruções para entrar na lista, sair da lista e usar a lista em
http://www.mat.puc-rio.br/~nicolau/olimp/obm-l.html
=



-
Novo Yahoo! Cadê? - Experimente uma nova busca.


_
MSN Messenger: instale grátis e converse com seus amigos. 
http://messenger.msn.com.br


=
Instruções para entrar na lista, sair da lista e usar a lista em
http://www.mat.puc-rio.br/~nicolau/olimp/obm-l.html
=


RE: [obm-l] Cone Sul 88

2007-07-14 Por tôpico Antonio Neto

Obrigado, Ponce. Abracos, olavo.



From: Rogerio Ponce [EMAIL PROTECTED]
Reply-To: obm-l@mat.puc-rio.br
To: obm-l@mat.puc-rio.br
Subject: [obm-l] Cone Sul 88
Date: Fri, 13 Jul 2007 22:19:12 -0300 (ART)

Oi Olavo,
temos que
 a**2 +  = b**2

Portanto,
 (b+a) * (b-a) = 
que pode ser decomposto em 11*101 ou em 1*

No primeiro caso,
(b+a)+(b-a) = 112 , de onde b=56 e a=45

No segundo caso,
(b+a)+(b-a) = 1112, de onde b=556 e a=555

Entretanto, no segundo caso, o numero a**2 tem mais que 4 algarismos.
Sobra apenas a primeira solucao, com a**2=2025 e b**2=3136

[]'s
Rogerio Ponce

-

Ola, amigos da lista, andei meio doente e sumido, mas sobrevivi. Enquanto 
estava de cama, andei vendo umas olimpiadas antigas, para me distrair e 
achei o seguinte problema: queremos um numero de 4 algarismos, todos 
menores que 6, e ao acrescentarmos 1 a todos os seus algarismos, obtemos 
outro quadrado perfeito. Achei 45^2 = 2025, e acrescentando 1 a todos os 
algarismos vem 3136 = 56^2. Mas achei a minha solucao muito bracal, alguem 
teria algo melhor, alguma propriedade de teoria dos números que eu nao 
saiba, ou nao lembrei?

Abracos, olavo.






-
Novo Yahoo! Cadê? - Experimente uma nova busca.


_
MSN Messenger: instale grátis e converse com seus amigos. 
http://messenger.msn.com.br


=
Instruções para entrar na lista, sair da lista e usar a lista em
http://www.mat.puc-rio.br/~nicolau/olimp/obm-l.html
=


[obm-l] Errata

2007-07-14 Por tôpico Antonio Neto

Desculpem, digitei errado. Vai a correcao:

queremos um numero de 4
algarismos, QUADRADO PERFEITO, todos menores que 6, e ao acrescentarmos 1 a 
todos os

seus algarismos, obtemos outro quadrado perfeito. Achei 45^2 = 2025,
e acrescentando 1 a todos os algarismos vem 3136 = 56^2.




From: Antonio Neto [EMAIL PROTECTED]
Reply-To: obm-l@mat.puc-rio.br
To: obm-l@mat.puc-rio.br
Subject: [obm-l] Cone Sul 88
Date: Fri, 13 Jul 2007 23:11:30 +


  Ola, amigos da lista, andei meio doente e sumido, mas sobrevivi. 
Enquanto estava de cama, andei vendo umas olimpiadas antigas, para me 
distrair e achei o seguinte problema: queremos um numero de 4 algarismos, 
todos menores que 6, e ao acrescentarmos 1 a todos os seus algarismos, 
obtemos outro quadrado perfeito. Achei 45^2 = 2025, e acrescentando 1 a 
todos os algarismos vem 3136 = 56^2. Mas achei a minha solucao muito 
bracal, alguem teria algo melhor, alguma propriedade de teoria dos números 
que eu nao saiba, ou nao lembrei? Abracos, olavo.


_
MSN Messenger: instale grátis e converse com seus amigos. 
http://messenger.msn.com.br


=
Instruções para entrar na lista, sair da lista e usar a lista em
http://www.mat.puc-rio.br/~nicolau/olimp/obm-l.html
=


_
MSN Messenger: instale grátis e converse com seus amigos. 
http://messenger.msn.com.br


=
Instruções para entrar na lista, sair da lista e usar a lista em
http://www.mat.puc-rio.br/~nicolau/olimp/obm-l.html
=


[obm-l] Cone Sul 88

2007-07-13 Por tôpico Antonio Neto


  Ola, amigos da lista, andei meio doente e sumido, mas sobrevivi. Enquanto 
estava de cama, andei vendo umas olimpiadas antigas, para me distrair e 
achei o seguinte problema: queremos um numero de 4 algarismos, todos menores 
que 6, e ao acrescentarmos 1 a todos os seus algarismos, obtemos outro 
quadrado perfeito. Achei 45^2 = 2025, e acrescentando 1 a todos os 
algarismos vem 3136 = 56^2. Mas achei a minha solucao muito bracal, alguem 
teria algo melhor, alguma propriedade de teoria dos números que eu nao 
saiba, ou nao lembrei? Abracos, olavo.


_
MSN Messenger: instale grátis e converse com seus amigos. 
http://messenger.msn.com.br


=
Instruções para entrar na lista, sair da lista e usar a lista em
http://www.mat.puc-rio.br/~nicolau/olimp/obm-l.html
=


[obm-l] Ajuda (Polin�mios)

2007-05-01 Por tôpico Antonio Neto


  Vc jah perguntou isso antes, pensei que alguem tinha respondido, é isso 
mesmo, apenas o termo que se usa eh IDENTICAMENTE nulo. Abracos, olavo.




From: cleber vieira [EMAIL PROTECTED]
Reply-To: obm-l@mat.puc-rio.br
To: obm-l@mat.puc-rio.br
Subject: [obm-l]   Ajuda (Polinômios)
Date: Tue, 1 May 2007 17:22:59 -0300 (ART)

  Amigos gostaria da ajuda de vocês a fim de resolver esse problema pois 
não estou conseguindo enxergar a saída. A fim de situá-lo, esse faz parte 
dos exercícios testes do livro do Iezzi nº6, Complexos, Polinômios e 
Equações (pg 221, nº84).


  O polinômio P(x) = a_o x^3 + a_1 x^2 + a_2 x + a_3 se anula para 4 
valores distintos de x. Podemos concluir que:


  a) a_0 + a_1 + a_2 + a_3 = 1 c) a_0  a_1  a_2  a_3

  b) (a_0)*(a_1)*(a_2)*(a_3) = 4 d) a_0  a_1  a_2  a_3

  e) a_0 = a_1 = a_2 = a_3

  A minha suspeita de resposta é o item e), visto que P(x) tem grau no 
máximo 3, caso em que a_0 é diferente de zero, mas como P(x) tem 4 raízes, 
P(x) não poderia ter grau 3 logo P(x) teria que ser nulo, ou seja, a_0 = 
a_1 = a_2 = a_3 = 0 independente dos valores de x.


  Desde já muito obrigado.
  Cleber



 __
Fale com seus amigos  de graça com o novo Yahoo! Messenger
http://br.messenger.yahoo.com/


_
MSN Messenger: instale grátis e converse com seus amigos. 
http://messenger.msn.com.br


=
Instruções para entrar na lista, sair da lista e usar a lista em
http://www.mat.puc-rio.br/~nicolau/olimp/obm-l.html
=


Re: [obm-l] Malba Tahan

2007-04-24 Por tôpico Antonio Neto

Receio que em alguns ele tenha usado fatorial e/ou radicais. Abracos, olavo


From: Johann Peter Gustav Lejeune Dirichlet 
[EMAIL PROTECTED]

Reply-To: obm-l@mat.puc-rio.br
To: obm-l@mat.puc-rio.br
Subject: Re: [obm-l] Malba Tahan
Date: Tue, 24 Apr 2007 11:10:02 -0300

Bem, no caso do livro do Malba Tahan, só pode usar as 4 operações básicas e
parênteses...

Em 23/04/07, saulo nilson [EMAIL PROTECTED] escreveu:


pode usar uma base diferente e usar exponencial, ou so pode aparecer o
sinal das 4 operaçoes.





On 4/18/07, regis barros [EMAIL PROTECTED] wrote:

 gostaria quem leu o livro, O homem que calculava,  que descobrisse como
 calcular com 4 quatros o número 99. eu encontre uma solução e gostaria 
de

 ter outras.

 regis

 __
 Fale com seus amigos de graça com o novo Yahoo! Messenger
 http://br.messenger.yahoo.com/






--
Ideas are bulletproof.

V


_
MSN Messenger: instale grátis e converse com seus amigos. 
http://messenger.msn.com.br


=
Instruções para entrar na lista, sair da lista e usar a lista em
http://www.mat.puc-rio.br/~nicolau/olimp/obm-l.html
=


[obm-l] GA-2

2007-03-29 Por tôpico Antonio Neto


  Basta notar que ambos terao a mesma norma, e (faca a figura) o 
paralelogramo se transforma em losango. Abracos, olavo.




From: carlos martins martins [EMAIL PROTECTED]
Reply-To: obm-l@mat.puc-rio.br
To: obm-l@mat.puc-rio.br
Subject: [obm-l] GA-2
Date: Wed, 28 Mar 2007 20:59:11 -0300

Mais uma de G.A.

Sejam u=AB e v=AC, vetores não nulos de normas p e q, respectivamente. 
Prove que o vetor w=qu+pv é paralelo à bissetriz de BÂC.


Obrigado.

_
Descubra como mandar Torpedos do Messenger para o celular! 
http://mobile.msn.com/


=
Instruções para entrar na lista, sair da lista e usar a lista em
http://www.mat.puc-rio.br/~nicolau/olimp/obm-l.html
=


_
MSN Messenger: instale grátis e converse com seus amigos. 
http://messenger.msn.com.br


=
Instruções para entrar na lista, sair da lista e usar a lista em
http://www.mat.puc-rio.br/~nicolau/olimp/obm-l.html
=


[obm-l] teoria dos n�meros - simples

2007-03-22 Por tôpico Antonio Neto
Note que todos os restos são cinco unidades a menos que os divisores. Se x 
eh o numero procurado, x + 5 eh o mmc dos divisores. Abracos, olavo.




From: saulo nilson [EMAIL PROTECTED]
Reply-To: obm-l@mat.puc-rio.br
To: obm-l@mat.puc-rio.br
Subject: Re: [obm-l] teoria dos números - simples
Date: Wed, 21 Mar 2007 23:56:27 -0300

pelas igualdades da para ver que q3 e um multiplo de 12 e de 3 , isso
minimiza o numero de testes.

On 3/21/07, saulo nilson [EMAIL PROTECTED] wrote:


q3=(5q1-7)/12=(2q2-1)/3
era so continuar testando 108 vinhaum pouco  depois de 48, nao testei o
q2.


 On 3/21/07, saulo nilson [EMAIL PROTECTED] wrote:

 M=10*q1+5
 M=16*q2+11
 M=24*q3+19
 24q3+8=16q2
 3q3+1=2q2
 24q3+14=10q1
 16q2+6=10q1

 3q3+1=2q2
 12q3+7=5q1
 8q2+3=5q1
 sistema acima e SPD a 1a equaçao e CL de 2 e 3
 24q3+8=10q1-6
 24q3=10q1-14
 q3=(5q1-7)/12
 omenor valor de q1 que torna q3 inteiro positivo
 5q1-7=48
 q1=11
 M=116


 On 3/21/07, Emanuel Valente [EMAIL PROTECTED]  wrote:
 
  Travei nessa questão. Agora é com vocês, cabeças.
  Determinar o menor número que dividido por 10; 16 e 24 deixa,
  respectivamente os restos 5; 11 e 19.
 
 
  
=

  Instruções para entrar na lista, sair da lista e usar a lista em
  http://www.mat.puc-rio.br/~nicolau/olimp/obm-l.html
 
  
=

 





_
MSN Messenger: instale grátis e converse com seus amigos. 
http://messenger.msn.com.br


=
Instruções para entrar na lista, sair da lista e usar a lista em
http://www.mat.puc-rio.br/~nicolau/olimp/obm-l.html
=


RE: [obm-l] Web Site de Geometria

2006-12-03 Por tôpico Antonio Neto
  Um colega me passou este, e achei interessante. Achei 11, mas não vou 
postar a solução para que o pessoal possa pensar mais. Abraços, olavo.




From: Luís Lopes [EMAIL PROTECTED]
Reply-To: obm-l@mat.puc-rio.br
To: obm-l@mat.puc-rio.br
Subject: RE: [obm-l] Web Site de Geometria
Date: Wed, 29 Nov 2006 15:32:43 +

Sauda,c~oes,

Oi Claudio,

Eu já conhecia este site, ele é mesmo muito legal e
bem feito.

Me passaram o seguinte problema, parece que de um
concurso pra Escola de Sargentos.

Num triângulo, b=12, c=10 e os pontos G e I estão
numa mesma reta paralela ao lado BC. Quanto vale a?

Há 5 escolhas de resposta mas deixo assim mesmo.

[]'s
Luís



From: claudio\.buffara [EMAIL PROTECTED]
Reply-To: obm-l@mat.puc-rio.br
To: obm-l obm-l@mat.puc-rio.br
Subject: [obm-l] Web Site de Geometria
Date: Tue, 28 Nov 2006 22:18:30 -0300

Oi, pessoal:

Achei um site muito legal sobre geometria, com applets contendo 
demonstracoes de varios teoremas classicos alem de alguns outros dos

quais eu nunca tinha ouvido falar. Vale a pena conferir.

http://agutie.homestead.com/files/geometry_help_online.htm

[]s,
Claudio.


_
MSN Messenger: converse com os seus amigos online. 
http://messenger.msn.com.br


=
Instruções para entrar na lista, sair da lista e usar a lista em
http://www.mat.puc-rio.br/~nicolau/olimp/obm-l.html
=


_
MSN Messenger: instale grátis e converse com seus amigos. 
http://messenger.msn.com.br


=
Instruções para entrar na lista, sair da lista e usar a lista em
http://www.mat.puc-rio.br/~nicolau/olimp/obm-l.html
=


[obm-l] Livro

2006-08-22 Por tôpico Antonio Neto

Tenho tres solucoes:

1 - Na www.doverpublications.com

2 - Se vc for do Rio de Janeiro, a Interciencia traz para cá, talvez para a 
sua cidade, cobrando o frete, é claro. Av Pres. Vargas, 435, 18 andar, 
(0xx21) 22 21 09 93.


3 - A MF importa tambem, mas fechou a loja do Rio, mas se vc for de SP eh a 
melhor pedida.


  Soh tenho uma pergunta, este livro ainda estah sendo editado? O melhor 
seria entrar na Dover e verificar, se nao quiser comprar com eles anota pelo 
menos o ISBN, que facilita a busca das livrarias especializadas. Abraco, 
olavo.




From: Douglas Ribeiro Silva [EMAIL PROTECTED]
Reply-To: obm-l@mat.puc-rio.br
To: obm-l@mat.puc-rio.br
Subject: [obm-l] Livro
Date: Mon, 21 Aug 2006 22:09:04 -0300

Alguem sabe onde eu posso adquirir o livro Advanced Euclidean
Geometry de R.A. Johnson, Dover Publications, 1960 ?


_
MSN Messenger: instale grátis e converse com seus amigos. 
http://messenger.msn.com.br


=
Instruções para entrar na lista, sair da lista e usar a lista em
http://www.mat.puc-rio.br/~nicolau/olimp/obm-l.html
=


[obm-l] Caminhada numa Esfera

2006-08-12 Por tôpico Antonio Neto

Aliás, não enumerável.



From: Guilherme Neves [EMAIL PROTECTED]
Reply-To: obm-l@mat.puc-rio.br
To: obm-l@mat.puc-rio.br
Subject: Re: [obm-l] Caminhada numa Esfera
Date: Fri, 11 Aug 2006 19:59:00 -0200


Tome uma circunferencia com 10km de comprimento ( ou simplesmente r=5/pi 
km) paralela ao equador no hemisfério sul. Suba agora 10 km para o norte e 
comece o seu problema. Você anda 10 km pro sul e chega na tal 
circunferencia paralela ao equador com 10km de comprimento. Quando você 
andar 10 km para o leste, você dará uma volta na esfera e chegará ao mesmo 
ponto. Andando 10km para o norte voce volta para o ponto inicial do 
problema.!! Tendo assim uma infinidade de soluções.


=
Instruções para entrar na lista, sair da lista e usar a lista em
http://www.mat.puc-rio.br/~nicolau/olimp/obm-l.html
=


_
MSN Messenger: instale grátis e converse com seus amigos. 
http://messenger.msn.com.br


=
Instruções para entrar na lista, sair da lista e usar a lista em
http://www.mat.puc-rio.br/~nicolau/olimp/obm-l.html
=


[no subject]

2006-07-25 Por tôpico Antonio Neto
i) Vai na marra, a paralela é 3x - 4y + C = 0, e a distância é (6 + 4 + 
C)/sqrt(9 + 16) = 2, ou C = 5.
ii) A perpendicular é 2x + y + C = 0. Pense o motivo. Ou vc põe a distância 
do centro (0, 0) à reta igual a 10, ou (mais geral), faz a interseção 
(sistema) e impõe delta = 0, processo que funciona com outras cônicas para 
achar tangentes.
iii) Seja a reta y = mx + h. Se ela passa pelo ponto (7, 3), temos 3 = 7m + 
h, e h = 3 - 7m, logo a reta é y = mx + 3 - 7m. Faz a interseção com as duas 
retas e impõe que o ponto médio seja (7, 3).

  Espero ter ajudado, abraços, olavo.



From: Henrique Ren [EMAIL PROTECTED]
Reply-To: obm-l@mat.puc-rio.br
To: obm-l@mat.puc-rio.br
Date: Wed, 26 Jul 2006 01:43:33 +

pessoal boa noite...

tenho alguns exercícios de g.a., vcs poderiam me ajudar?

i) achar a reta paralela a 3x-4y+4=0, cuja dist. do pto p (2,-1) é 2
ii) achar a reta perpendicular a 2x-y=0 e tangente a  x^2 + y^2=100
iii) obter a reta q intercepta x-2y=0 e x-4y=0 em dois pontos A e B, tais q 
o ponto médio de AB é M(7,3)


ah, o baricentro de um triangulo de vértices A(xa,ya), B(xb,yb) e C(xc,yc), 
é Xg= (xa+xb+xc)/3 e Yg= (ya+yb+yc)/3.  como acho o incentro ], 
circuncentro e orocentro?


obrigadão.


=
Instruções para entrar na lista, sair da lista e usar a lista em
http://www.mat.puc-rio.br/~nicolau/olimp/obm-l.html
=


_
MSN Messenger: instale grátis e converse com seus amigos. 
http://messenger.msn.com.br


=
Instruções para entrar na lista, sair da lista e usar a lista em
http://www.mat.puc-rio.br/~nicolau/olimp/obm-l.html
=


RE: [obm-l] Ajuda

2006-07-18 Por tôpico Antonio Neto

a - 2b = - 7. Abraços, olavo.



From: Sharon Guedes [EMAIL PROTECTED]
Reply-To: obm-l@mat.puc-rio.br
To: obm-l@mat.puc-rio.br
Subject: [obm-l] Ajuda
Date: Tue, 18 Jul 2006 00:15:48 -0300 (ART)

Olá pessoal, alguém poderia me ajudar a resolver essa questão? Resolvi ela 
mas quero ter certeza da resposta.


  A função y = ax + b passa pelo ponto (1, 2) e intercepta o eixo y no 
ponto de ordenada 3. Então a - 2b é igual a ?


  Um abraço Sharon


-
 Novidade no Yahoo! Mail: receba alertas de novas mensagens no seu 
celular. Registre seu aparelho agora!


_
MSN Messenger: instale grátis e converse com seus amigos. 
http://messenger.msn.com.br


=
Instruções para entrar na lista, sair da lista e usar a lista em
http://www.mat.puc-rio.br/~nicolau/olimp/obm-l.html
=


[obm-l] Equa��o da Reta

2006-07-16 Por tôpico Antonio Neto
Naturalmente que não pode usar calculadora, em vestibulares não é permitido. 
Há duas maneiras de obter as bissetrizes dos ângulos entre duas retas 
concorrentes dadas:
1 - Utilizando vetores normais. Das equações, você tem os vetores normais às 
retas. Calcule seus módulos, e os multiplique (ou divida, o que é o mesmo) 
por números convenientes, para que tenham o mesmo módulo. Isto garante que a 
sua soma estará sobre a sua bissetriz, já que o paralelogramo se transforma 
em losango. Então esta soma é normal a uma das bissetrizes. Determine a reta 
que tem este vetor normal e passa pela interseção das retas dadas. A outra 
bissetriz é a perpendicular à primeira e que passa pela mesma interseção. 
Não recomendo, exceto se os números forem amigáveis (não confundir cim 
números amigos, que é outra coisa), por exemplo, os vetores normais forem 
(3, 4), (5, 12), (7, 24), isto é, tiverem módulos inteiros (catetos de 
triângulos retângulos de lados inteiros).
2 - A bissetriz de um ângulo é o lugar geométrico dos pontos equidistantes 
dos seus lados, e isto dá a sulução simples da coisa. Seja P(x, y) um ponto 
genérico da bissetriz, e utilize a fórmula da distância de ponto a reta. 
Sejam dadas as retas Ax + By + C = 0 e Dx + Ey + F = 0. Teremos
|(Ax + By + C)/sqrt(A^2 + B^2)| = |(Dx + Ey + F/sqrt(D^2 + E^2)|, e isso dá 
as duas bissetrizes mais rapidamente. No seu caso (faça o gráfico) a 
bissetriz deve estar situada nos quadrantes ímpares, logo deve ter 
coeficiente angular positivo.

Acrediti que isto possa ajudar, Abraços, olavo.



From: João Vitor [EMAIL PROTECTED]
Reply-To: obm-l@mat.puc-rio.br
To: obm-l@mat.puc-rio.br
Subject: [obm-l] Re: [obm-l] Equação da Reta
Date: Sun, 16 Jul 2006 16:16:40 -0300

kra,
eu faria assim
Eq1: y = 0 (reta coincidente com o eixo X) tg H = 0 = H=0
Eq2: y = 3x (reta com coef angular = 3, ou seja forma um cuja tangente é 
igual a 3 com o eixo x) - tg K = 3 - K=arc tg 3

k= 71, 5650°

o angulo da bissetria seria a média aritmética entre H e K
A = (h+k)/2   - A = 35, 78°

eq geral da reta:

Y = Mx + N   - N=0
Y=Mx

onde tg 35, 78º = M = 0, 7206

eq da bissetriz é Y = 0,7206X

pode usar calculadora lá?
flw



- Original Message - From: João Carlos [EMAIL PROTECTED]
To: OBM obm-l@mat.puc-rio.br
Sent: Saturday, July 15, 2006 9:47 AM
Subject: [obm-l] Equação da Reta



Esta questão caiu na 2ºEtapa do vestibular da UFMG de
2002, não estou conseguindo resolver.

Determine a equação da bissetriz do menor ângulo
formado pelas retas de equações y=0 e y=3x.

Desde já agradeço pela ajuda de vocês.


 /  \ /| |'-.
.\__/ || |   |
 _ /  `._ \|_|_.-'
| /  \__.`=._) (_   Júnior
|/ ._/  ||
|'.  `\ | | Desenvolvedor de Softwares
;/ / | | Seja Livre - Use Linux
) /_/|  |.---.| E-mail:[EMAIL PROTECTED]
'  `-`  ' Msn:[EMAIL PROTECTED]




___
Novidade no Yahoo! Mail: receba alertas de novas mensagens no seu celular. 
Registre seu aparelho agora!

http://br.mobile.yahoo.com/mailalertas/


=
Instruções para entrar na lista, sair da lista e usar a lista em
http://www.mat.puc-rio.br/~nicolau/olimp/obm-l.html
=



=
Instruções para entrar na lista, sair da lista e usar a lista em
http://www.mat.puc-rio.br/~nicolau/olimp/obm-l.html
=


_
MSN Messenger: instale grátis e converse com seus amigos. 
http://messenger.msn.com.br


=
Instruções para entrar na lista, sair da lista e usar a lista em
http://www.mat.puc-rio.br/~nicolau/olimp/obm-l.html
=


RE: [obm-l] Produto Vetorial

2006-07-16 Por tôpico Antonio Neto
  Henrique, se a sua faculdade tiver um mínimo de decência, terá exemplares 
do livro do Natan Moreira dos Santos, Vetores e Matrizes. Não sei se ainda 
está sendo editado, mas deveria. É uma excelente introdução à Álgebra 
Linear, para quem está entrando no assunto, e tem uma definição mais 
intuitiva de produto vetorial




From: Henrique Rennó [EMAIL PROTECTED]
Reply-To: obm-l@mat.puc-rio.br
To: obm-l@mat.puc-rio.br
Subject: [obm-l] Produto Vetorial
Date: Sun, 16 Jul 2006 20:25:27 -0300

Olá!!!

Gostaria de saber se alguém poderia dar uma demonstração de como são
definidos os componentes de um vetor perpendicular a outros dois
vetores utilizando o produto vetorial em três dimensões. Eu sei que é
necessário calcular o determinante dos dois vetores da seguinte forma
para achar o vetor:

|  i j   k |
| x1 y1 z1 |  =  (y1z2) i + (x2z1) j + (x1y2) k - (y2z1) i - (x1z2) j
- (x2y1) k.
| x2 y2 z2 |

Dessa forma as componentes do vetor resultante serão:

(y1z2 - y2z1 , x2z1 - x1z2 , x1y2 - x2y1).

Mas como pode ser demonstrada essa relação entre o determinante e o
vetor perpendicular???

A necessidade dessa demonstração surgiu quando precisei calcular a
área entre dois vetores de duas dimensões.

Representei os dois vetores u e v num sistema x,y e calculei a área do
paralelogramo formado por eles como |u|.|v|.sen(c), onde c é o
ângulo entre os dois vetores. Depois representei c = b - a, em que b
e a são os ângulos entre os vetores v e u, respectivamente, e o eixo
x.

Utilizando a fórmula sen(c) = sen(b-a) = sen(b)cos(a) - cos(b)sen(a)
achei a área do paralelogramo como ux.vy - uy.vx, que é o determinante
entre a matriz composta pelos componentes de u e v.

| ux uy |
| vx vy  |

Estendendo para três dimensões não sei como demonstrar o produto
vetorial, o qual em vários livros já é dado definido como mencionei
acima e que essa operação entre vetores fornece um vetor perpendicular
aos dois vetores sobre o qual foi calculado.

Também li que o produto escalar entre o vetor perpendicular resultante
e um outro vetor w diferente de u e v (em que w tem a origem
coincidindo com origem de u e v) fornece o volume do sólido formado
pelos vetores u, v e w.

Essa definição consegui mostrar utilizando a base inferior e superior
do sólido e utilizando o cosseno entre o vetor w e um outro vetor
perpendicular que formam um ângulo d calculei a altura do sólido
como |w|.cos(d).

Volume = área da base * altura = | u x v | * | w | * cos(d), que é o
produto escalar entre o vetor u x v e w. Volume = | (u x v) . w |

A igualdade cos(theta) = (A.B) / |A||B| pode ser verificada através do
cosseno da diferença de dois ângulos.

Assim, se puder existir uma demonstração simples e clara do produto
vetorial ficarei muito grato. Estou estudando Álgebra Linear e os
livros que peguei na faculdade em nível de Graduação não são tão
didáticos, sendo que os autores consideram que o leitor já tenha
conhecimento de diversos conceitos para várias fórmulas que são apenas
dadas. É bem diferente de livros utilizados no Ensino Médio.

Pediria também alguma indicação para livros sobre o assunto Álgebra
Linear. Sempre estudo matemática e adoro a rainha das ciências, mas
agora vejo que o nível de abstração está ficando cada vez maior.

Grato pela atenção,

--
Henrique

=
Instruções para entrar na lista, sair da lista e usar a lista em
http://www.mat.puc-rio.br/~nicolau/olimp/obm-l.html
=


_
On the road to retirement? Check out MSN Life Events for advice on how to 
get there! http://lifeevents.msn.com/category.aspx?cid=Retirement


=
Instruções para entrar na lista, sair da lista e usar a lista em
http://www.mat.puc-rio.br/~nicolau/olimp/obm-l.html
=


Re: [obm-l] Tri�ngulos Pitag�ricos (was:12^2 + 33^2 = 1233^2)

2006-06-13 Por tôpico Antonio Neto

  Amigos, boa noite,

  na verdade, o que eu acho mais eficaz, e que já é conhecido desde 
Euclides, é: sejam m e n inteiros positivos, com mn. Entao, m^2 - n^2, 2mn 
e m^2 + n^2 será sempre um triângulo pitagórico, com o inconveniente de 
fornecer repetições (triângulos semelhantes). Para eliminar estes casos 
repetidos, acrescente que m e n devem ter paridades diferentes e devem ser 
primos entre si. Aí, dá tudo certo. A ida da demonstração é óbvia, produtos 
notáveis, a volta é mais interessante. Abraços, olavo.



From: Johann Peter Gustav Lejeune Dirichlet 
[EMAIL PROTECTED]

Reply-To: obm-l@mat.puc-rio.br
To: obm-l@mat.puc-rio.br
Subject: Re: [obm-l] Triângulos Pitagóricos (was:12^2 + 33^2 = 1233^2)
Date: Tue, 13 Jun 2006 12:09:54 -0300

Desculpe informar mas a formula ai escrita nao serve (acho) para todos os
triangulos pitagoricos. Sempre tem algum que escapa.
Para capturar todos eles e necessario usar pelo menos umas duas variaveis
livres. Se eu nao me engano a formula
(u^2-v^2)^2+(2uv)^2=(u^2+v^2)^2
serve, com alguns inconvenientes de produzir numeros repetidos.

Em 09/06/06, [EMAIL PROTECTED] [EMAIL PROTECTED] escreveu:



Oi pessoal, vamos acalmar com calma:
   Espero que essa mensagem possa ajudar neste problema (embora
possa como todas as minhas outras possa
ser apenas um pitaco sem nenhuma utilidade).

 Sabemos que:
 (n^2 - 1)^2 + (2n)^2 = (n^2 +1)^2

   para n natural, n1 ela dá todos os triângulos pitagóricos.
   Ex: n=2 : 3^2 + 4^2 = 5^2 .
A intenção é usar essa identidade para tentar obter quadrados
perfeitos naturais da forma Delta^2 = b^2 - 4ac.
   Neste caso usamos:
  (n^2 - 1)^2  = (n^2 +1)^2  - (2n)^2
(n^2 - 1)^2 = (n^2 +1)^2 - 4 n^2

Supondo a = 1 (sempre dá para fazer a=1 em uma eq. do 2 grau).
  Temos então que ter:
b = n^2 +1
c= n^2   == b = c+1

 Bom... agora será que dá para aplicar isso à equação em jogo?

100a+b = a^2 + b^2
basta resolver essa eq de 2º grau com relação a a
e temos
a = 50 +- sqrt(2500+b-b^2)

   Para não causar confusão vamos trocar a por x e b por y:

   100x + y = x^2 + y^2

   x^2 -100x +y -y^2 = 0

   Construindo o Delta:
Delta^2 = 100^2 - 4*(y-y^2)

 com b = 100 e c = y-y^2
 como b= c+1
 100 = y-y^2 +1

   Quais y naturais com 2 algarismos verificam isso?




=
Instruções para entrar na lista, sair da lista e usar a lista em
http://www.mat.puc-rio.br/~nicolau/olimp/obm-l.html
=





--
Ideas are bulletproof.

V


_
MSN Messenger: instale grátis e converse com seus amigos. 
http://messenger.msn.com.br


=
Instruções para entrar na lista, sair da lista e usar a lista em
http://www.mat.puc-rio.br/~nicolau/olimp/obm-l.html
=


[obm-l] Integral e coisas tiradas do bolso...

2005-06-13 Por tôpico Antonio Neto

 Amigos da lista,
 este é um terreno movediço, e devo lembrar que trabalho com Ensino Medio, que apresenta certas peculiaridades, mas cabe citar Polya, que (procurei a citacao, mas não achei) escreveu "a trick is a device you use twice", acho que é isso. A nivel de Ensino Medio, isso significa liberar os alunos de decorar a formula da bissetriz interna relativa ao lado a de um triangulo ( dados os lados a, b e c) e utilizar a lei dos co-senos duas vezes, junto com o teorema da bissetriz. Acredito que seja mais produtivo, em termos de raciocinio, do que ficar decorando as formulas para os comprimentos de alturas, medianas, bissetrizes (internas e externas) e cevianas em geral. Claro que isso é só um exemplo, procuro melhorar as minhas aulas de ano para ano, e recomendo aos outros professores do Ensino Medio da lista o Polya e o Gelfand. Fora os livros da minha epoca, que agora so nos sebos. 
Ou, sem modestia, aqui em casa. abracos, olavo.From: "Nicolau C. Saldanha" [EMAIL PROTECTED]Reply-To: obm-l@mat.puc-rio.brTo: obm-l@mat.puc-rio.brSubject: Re: [obm-l] Integral e coisas tiradas do bolso...Date: Mon, 13 Jun 2005 10:11:31 -0300Qual é exatamente a diferença entre uma idéia genial, um truque, um macetee um método? Para mim é uma diferença de ponto de vista. Quem descobriua idéia e dela se orgulha a considera uma idéia genial. Os amigos delequando veem a idéia chamam aquilo de truque, com um misto de admiraçãoe inveja. Os adversários que não tiveram a idéia certa na hora certapor despeito chamam de macete. Depois que a idéia já foi usada maisde três vezes ela vira um método. Depois de ser usada 100 vezes a idéiajá não tem mais nome nenhum, 
passa a ser considerada uma trivialidadeindigna de ser mencionada. :-)[]s, N.=Instruções para entrar na lista, sair da lista e usar a lista emhttp://www.mat.puc-rio.br/~nicolau/olimp/obm-l.html=MSN Messenger: converse com os seus amigos online. Instale grátis. Clique aqui. 

=
Instruções para entrar na lista, sair da lista e usar a lista em
http://www.mat.puc-rio.br/~nicolau/olimp/obm-l.html
=


[obm-l] quest�o de geometria

2005-04-30 Por tôpico Antonio Neto

 Amigos,
 acredito que a questão se refira ao número de pontos de interseção interiores ao polígono. No caso, cada escolha de quatro vértices dá origem a um quadrilátero convexo, cujas diagonais se intersectam em um ponto interior. Logo, o número procurado é o mesmo número de quadriláteros convexos, ou seja, bin(n, 4). Abraços, olavo.
From: "Brunno Fernandes" [EMAIL PROTECTED]
Reply-To: obm-l@mat.puc-rio.br
To: obm-l@mat.puc-rio.br
Subject: [obm-l] Re: [obm-l] questão de geo
Date: Sat, 30 Apr 2005 10:09:10 -0300

Ola Daniel tudo bem??
Obrigado pela força

Nesta questãoo gabarito indica
[n x (n-1) x (n-2) x (n-3)]/ (1x2x3x4)

Estou tentando achar erros mas não estou conseguindo
Vou analisar com mais calma, mas queria já te mandar o gabarito e assim você
pode já ver se tem algum erro
Um abraço Daniel
Do amigo
Brunno


- Original Message -
From: [EMAIL PROTECTED]
To: obm-l@mat.puc-rio.br
Sent: Friday, April 29, 2005 1:42 AM
Subject: Re: [obm-l] questão de geo


Oi,
Eu acho que cheguei na resposta. A idéia é a seguinte:

De cada ponto partem (n - 3) diagonais, logo são d = n*(n-3)/2 diagonais no
total. Para determinar o número máximo de interseções, consideramos a melhor
das hipóteses: três diagonais distintas não se interceptam num mesmo ponto a
menos que se trate de um vértice do polígono. Ou seja, fixada uma diagonal,
quaisquer outras duas diagonais cortam a primeira em pontos distintos se
estas duas novas diagonais se não cruzam num vértice da primeira.

Dito isto, para o cálculo do número de pontos de interseção, imaginamos
inicialmente que quaisquer duas diagonais interceptam-se sempre em pontos
distintos. Então seriam d*(d-1)/2 interseções.

Só que, na verdade, para cada vértice, existem (n-3) diagonais que se
interceptam nele. Ou seja, até aqui estamos contando (n-3)*(n-4)/2
interseções ao invés de uma para cada vértice. Para corrigir o problema,
devemos tirar de d*(d-1)/2 as n*((n-3)*(n-4)/2 - 1) interseções contadas a
mais. Assim, o número máximo de interseções é

d*(d-1)/2 - n*((n-3)*(n-4)/2 - 1) =
= n*(n-3)*[n*(n-3)/2 - 1]/4 - n*[(n-3)*(n-4)/2 - 1].

OBS 1: só vale para n = 5... Quando n = 4, não existem duas diagonais
saindo do mesmo vértice, por isso fica somente d*(d-1)/2 = 1.
OBS 2: as diagonais podem interceptar-se fora do polígono!

[]s,
Daniel

Brunno Fernandes ([EMAIL PROTECTED]) escreveu:
 
 Ola pessoal tudo bem?
 Poderiam me ajudar nesta questão,
 
 Determinar o numero máximo de pontos de intersecção das diagonais de um
 poiligono convexo de n lados
 
 Uma questão muito parecida em que pede o número máximo de pontos de
 intersecção dos prolongamentos das diagonais
 
 Essas são questões do livro de Geometria Plana do livro do Edgardde
 Alencar Filho
 um ótimo livro
 Um abraço
 Do amigo
 Brunno

=
Instruções para entrar na lista, sair da lista e usar a lista em
http://www.mat.puc-rio.br/~nicolau/olimp/obm-l.html
=

=
Instruções para entrar na lista, sair da lista e usar a lista em
http://www.mat.puc-rio.br/~nicolau/olimp/obm-l.html
=
MSN Messenger: converse com os seus amigos online. Instale grátis. Clique aqui. 

=
Instruções para entrar na lista, sair da lista e usar a lista em
http://www.mat.puc-rio.br/~nicolau/olimp/obm-l.html
=


[obm-l] Por 7

2005-04-14 Por tôpico Antonio Neto

 Soh para podermos mudar de assunto, a minha fonte foi (na edição que tenho na bisbilhoteca) o *Criterios de divisibilidad* (em espanhol) de N. N. Vorobiov. Da Mir, naturalmente. Claro que há outros criterios, baseados em congruencias, como o das classes, mas estao explicados em livros de teoria dos numeros, e nao eh dificil achar. Soh mencionei porque eh pouco conhecido (acho), e eh curioso. A proposito, um aluno meu, de uma turma IME-ITA, que tem um fraco por essas coisas, desenvolveu um louco criterio de divisibilidade, que tem duncionado na pratica, mas ainda nao demonstrei. Se nao conseguir no fim de semana, que eh quando posso trabalhar (quem dah aula nao trabalha), mando pra voces. Abracos, olavo.
From: João Gilberto Ponciano Pereira [EMAIL PROTECTED]
Reply-To: obm-l@mat.puc-rio.br
To: "'obm-l@mat.puc-rio.br'" obm-l@mat.puc-rio.br
Subject: RE: [obm-l] Por 7
Date: Thu, 14 Apr 2005 11:14:25 -0300

Opa... peraí... Pelo que entendi, a regra valeria apenas para um dígito.

Veja o caso de 59768758231 (que é divisível por 7)

5976875 - 2*8231 = 5960413
596 - 2*413 = -230 (que não é divisível por 7!!!)

Isso pq essa regra funciona pois 2*10 mod 7 = -1.

Para funcionar para números "grandes", podemos usar que 1000 mod 7 = -1.
Assim, usaríamos 3 dígitos, agilizando as contas, e não seria nem necessário
multiplicar por 2.

o mesmo exemplo, teríamos:
59768758 - 231 = 59768527
59768- 527 = 59241
59 - 241 = -182, que é divisível.

SDS
JG


-Original Message-
From: Johann Peter Gustav Lejeune Dirichlet
[mailto:[EMAIL PROTECTED]
Sent: Monday, April 11, 2005 3:48 PM
To: obm-l@mat.puc-rio.br
Subject: RE: [obm-l] Por 7


--Que historia e essa de "so e bom para numeros
pequenos"? Para comeco de historia, pode-se pegar
qualquer bloco de digitos em vez de apenas um por vez.
Ou seja, podemos fazer algo como:

5976875(8234)
- 16468
-
5960407

 596(0407)
-0407
-0407
-218

E isto nao e multiplo de 7.
(So para desencargo de consciencia, conferi na BC...)

E apenas para terminar, normalmente ninguem te
perguntaria se o numero abaixo e ou nao multiplo de 7:

597687582345976875823459768758234597687582 \
345976597687582345976875823459768758234597 \
687582345976597687582345976875823459768758 \
234597687582345976597687582345976875823459 \
768758234597687582345976875823459768758234 \
598947895789456844566496313554564654456613 \
324165456489789754123164641304104817105130 \
152895531714012404504576875823459768758234 ^ 2

/*Os \ sao apenas quebras de linha para maior
legibilidade*/
/*Ou para menor ilegibilidade, entenda como
quiser...*/

Neste caso especificamente, nenhum criterio e melhor
que o outro (ou estou muito enganado, o que
ultimamente e um fato-comum)...

--- Rafael Alfinito Ferreira [EMAIL PROTECTED]
wrote:
  este processo só é bom para números pequenos, por
  exemplo:
  me diga se 59768758234 é divisível por 7 , aí é
  melhor usar a técnica das
  classes.
 
  VALEU! UM ABRAÇO!
  RAFAEL FERREIRA
 





Yahoo! Acesso Grátis - Internet rápida e grátis.
Instale o discador agora! http://br.acesso.yahoo.com/
=
Instruções para entrar na lista, sair da lista e usar a lista em
http://www.mat.puc-rio.br/~nicolau/olimp/obm-l.html
=

=
Instruções para entrar na lista, sair da lista e usar a lista em
http://www.mat.puc-rio.br/~nicolau/olimp/obm-l.html
=
MSN Messenger: converse com os seus amigos online. Instale grátis. Clique aqui. 

=
Instruções para entrar na lista, sair da lista e usar a lista em
http://www.mat.puc-rio.br/~nicolau/olimp/obm-l.html
=


[obm-l] Livro do Lidsky MIR

2005-03-28 Por tôpico Antonio Neto

 Sem machismo, acho que o livro eh DO Lidsky, penso que o autor eh homem, mas fui procurar e nao achei, o meu filho (que nao mora comigo) tem saqueado a bisbilhoteca nos dois ultimos anos, para fazer vestibular, e deve estar com ele. A sugestao eh procurar em sebos, tirar copia eh ilegal, e acho que quem le russo nalista eh o Paulo Santa Rita, mas tem que perguntar como anda o tempo dele. Se o meu tempo der, posso dar uma ajuda, o meu estah em ingles, mas tenho que acha-lo. Agora, se o livro estah disponivel na internet, eh verificar se eh livre, e a partir daih eh possivel traduzir. Desculpem, sei que eh off, foi soh para responder, qualquer correspondencia para o meu e-mail particular, estah aih em cima.Abracos, olavo.
From: André Barreto [EMAIL PROTECTED]
Reply-To: obm-l@mat.puc-rio.br
To: obm-l@mat.puc-rio.br
Subject: [obm-l] Livro da Lidsky MIR
Date: Mon, 28 Mar 2005 01:10:59 -0300 (ART)

Oi amigos da lista!

A algum tempo na lista o livro da Lidski foi indicado como uma excelente referência bibliográfica.

Tentei encontrar este livro para comprar, tirar cópia e etc... mas a minha empreitada foi sem sucesso.

Então pesquisando na internet encontrei uma versão dele digitalizada em russo. Não sei o russo.

Mas pensei que na lista alguém soubesse e tive-se interesse em traduzir ao menos os exercícios.

Porem me surgiu uma dúvida, creio que não seja proibido traduzir e distribuir os exercícios, pois acho que não se pode ter direitos autorais sobre estes, mas não tenho certeza.

Gostaria que alguém me informa-se, caso saiba, se o livro da Lidski ainda é comercializado ou está fora da circulação?

Caso alguém queira o livro eu posso enviar.

Obrigado

Atenciosamente,

André Sento Sé Barreto


__
Converse com seus amigos em tempo real com o Yahoo! Messenger
http://br.download.yahoo.com/messenger/
MSN Messenger: converse com os seus amigos online. Instale grátis. Clique aqui. 

=
Instruções para entrar na lista, sair da lista e usar a lista em
http://www.mat.puc-rio.br/~nicolau/olimp/obm-l.html
=


[obm-l] Exercicios

2005-03-19 Por tôpico Antonio Neto

 Oi, Daniele,
 nao cnsegui ler alguma parte dos exercicios, algo saiu errado, manda de novo? Beijo, carinho, olavo.
From: Daniela Yoshikawa [EMAIL PROTECTED]
Reply-To: obm-l@mat.puc-rio.br
To: obm-l@mat.puc-rio.br
Subject: [obm-l] Exerciacute;cios
Date: Sat, 19 Mar 2005 19:08:13 -0300 (ART)

Provar:

1) a + b = 0 ; a != 0; b != 0 - a/b^2 + b/a^2 = 1/a + 1/b
 != (diferente)

2) (a=0 b=0 c=0) - a + b + c = ãab + ãbc + ãac

3) (a=0 b=0 c=0) - ab(a+b) + bc(b+c) + ac(a+c) = 6abc

Muito obrigada,
Daniele.



-
Yahoo! Mail - Com 250MB de espaço.Abra sua conta!
MSN Messenger: converse com os seus amigos online. Instale grátis. Clique aqui. 

=
Instruções para entrar na lista, sair da lista e usar a lista em
http://www.mat.puc-rio.br/~nicolau/olimp/obm-l.html
=


[obm-l] sexta feira 13...

2005-02-23 Por tôpico Antonio Neto

 Oi,
 o Bernardo já deu uma resposta, só entrei poeque estava passando, geralmente chego atrasado, quem dá muita aula só lê as respostas. Mas é possível provar que todo ano, normal ou bissexto, tem no mínimo uma e no máximo três sextas-feiras 13. Há uma demonstração no Niven, "Mathematics of Choice", a edição que tenho é a da Random House, The L. W. Singer Company, quarta reimpressão, está logo na página 1, mas é o problema E1541 do American Mathematical Monthly, Nov 1962, pag 919, como o próprio Niven informa. Naturalmente, haverá mais soluções por aí, só estou tentando colaborar. Abraços, olavo.
From: "carlos gomes" [EMAIL PROTECTED]
Reply-To: obm-l@mat.puc-rio.br
To: obm-l@mat.puc-rio.br
Subject: [obm-l] sexta feira 13...
Date: Wed, 23 Feb 2005 08:27:49 -0300

Pessoal,
E verdade que todo ano tem pelo menos uma sexta-feira 13? Se for verdade como verifico isto?. Um abraço a todos Cgomes.
MSN Messenger: converse com os seus amigos online. Instale grátis. Clique aqui. 

=
Instruções para entrar na lista, sair da lista e usar a lista em
http://www.mat.puc-rio.br/~nicolau/olimp/obm-l.html
=


[obm-l] Alunos (era: Questão de conjuntos)

2005-01-26 Por tôpico Antonio Neto

 Amigo Artur,
talvez você tenha razão, mas é consenso entre os professores que os alunos já não são os mesmos de antigamente. O assunto é off-topic, mas é fácil entender que as motivações são outras, os apelos em maior número e variedade, etc. E veja que não posso reclamar muito, o lugar onde trabalho tem um bom conceito aqui no Rio de Janeiro. E é claro que existem as recompensas, já tive excelentes alunos, um deles foi parar na Microsoft, sempre que vinha ao Brasil me trazia gentilmente um livro de Matemática, outro foi para a Unicamp, onde se doutourou em Matemática Aplicada, e teve a gentileza de dedicar a mim a sua tese de mestrado. Estes e outros dizem que foram fazer Matemática por minha causa, o que me deixa muito gratificado, mas são minoria. Pensando bem, se todos fizessem Matemática, o que faríamos nós quando precisássemos de geriatras? :) Abraços a todos, olavo.
From: Artur Costa Steiner [EMAIL PROTECTED]
Reply-To: obm-l@mat.puc-rio.br
To: obm-l@mat.puc-rio.br
Subject: [obm-l] RES: [obm-l] Questão de conjuntos
Date: Tue, 25 Jan 2005 21:12:46 -0200

Serah que o meu amigo professor nao estah com uma visao um tanto pessimista
e radical? talvez o que aconteca eh que nem todos vibram com a matematica e
nem todos tem interesse em se aprofundar. Eu, por exemplo, sempre fui
apaixonado, mas por questoes praticas da vida nao pude me aprofundar como
queria. Assim, sempre que posso estudo mais os assuntos de que mais gosto,
dentre os quais inclui-se a Analise Matematica .Eu tambem curto muito
Geometria e Teoria dos Numeros, mas na falta de tempo, dou mais prioridade a
Analise. Com relacao a Geometria, eu na epoca de meu vestibular de
Engenharia resolvia problemas dificeis e, sem decorar, acabei guardadndo na
cabeca ateh hoje algumas formulas, como a da bisstriz interna dos lados de
um triangulo e tambem alguns teoremas. Mas hoje,em Geometria, eu no tenho
mais a agilidade que tinha no vestibular.
Artur

MSN Messenger: converse com os seus amigos online. Instale grátis. Clique aqui. 

=
Instruções para entrar na lista, sair da lista e usar a lista em
http://www.mat.puc-rio.br/~nicolau/olimp/obm-l.html
=


[obm-l] Questão de conjuntos

2005-01-25 Por tôpico Antonio Neto

Eu estava quietinho, mas tive na segunda o grande prazer de almoçar com amigos muito prezados, e comentamos esta mensagem. Acho, pela minha experiência como professor, que o aluno é como vinagre, piora com o tempo. Já tive alunos de oitava série que depois voltaram a ser meus no terceiro ano, e eles eram piores, não faziam mais nada em Geometria. Não tenha medo, leve os moleques a raciocinar, e eles vão responder. Só não seja como eu, que tenho que apanhá-los novamente no terceiro ano, e a esperança vai para o brejo. A propósito, os amigos eram o Morgado, o Wagner e o Paulo Cezar Pinto, e vourevê-los amanhã, com o prazer que a ocasião pede. Abraços, olavo.
From: "Nicolau C. Saldanha" [EMAIL PROTECTED]
Reply-To: obm-l@mat.puc-rio.br
To: obm-l@mat.puc-rio.br
Subject: Re: [obm-l] Re: [obm-l] Questão de conjuntos
Date: Tue, 25 Jan 2005 16:44:12 -0200

  Infelizmente não posso resolver usando analise combinatoria pois é para uma
  turma de 8ª série.
 
   C(8,5) = 8*7*6/(3*2) = 56 subconjuntos distintos de 5 elementos
   distintos a partir de um conjunto de 8 elementos distintos.
   
Gostaria de saber como resolver a seguinte questão:
   
Dado um conjunto com 8 elementos distintos, quantos subconjuntos com 5
elementos distintos podemos formar. (obs.: tenho que resolver usando
matemática de 1º grau).

A minha recomendação é que ao estudar análise combinatória,
as fórmulas sejam a última coisa a ser estudada e não a primeira.
Assim, eu resolveria este problema para alunos de 8a série
da seguinte maneira.

Num subconjunto os elementos não têm ordem.
Ao invés de contarmos conjunto, vamos primeiro contar listas
de 5 elementos distintos.

Vamos escolher o primeiro elemento da lista: temos 8 maneiras de fazer isso.
Vamos agora escolher o segundo elemento: temos 7 maneiras de fazer isso
pois um elemento já está tomado. Observe (isto é crucial) que temos *sempre*
7 maneiras de escolher o segundo elemento qualquer que tenha sido o primeiro.
Vamos escolher o terceiro: temos 6 maneiras. O quarto: 5 maneiras.
O quinto: 4 maneiras. Assim, temos 8*7*6*5*4 listas de 5 elementos.

Ora, cada conjunto corresponde a várias listas. Exatamente quantas?
Tantas quantas são as listas de 5 elementos que podemos fazer com 5 elementos.
Pelo mesmo raciocínio usado acima, temos 5 maneiras de escolher o primeiro
elemento da lista, 4 de escolher o segundo, 3 de escolher o terceiro,
2 de escolher o quarto e 1 (obviamente) de escolher o quinto.
Assim, podemos formar 5*4*3*2*1 listas de 5 elementos a partir
de um conjunto de 5 elementos. Em outras palavras,
cada conjunto de 5 elementos foi contado vezes.
Logo, o número de conjuntos é (8*7*6*5*4)/(5*4*3*2*1) = 56.

Eu evitei deliberadamente usar as palavras "fatorial", "permutação",
"arranjo" e "combinação". Acho que assim como as fórmulas, as palavras
"fatorial" e "permutação" só devem ser apresentadas ao aluno *depois*
que ele tenha entendido e feito sozinho problemas como este.
Quanto às palavras "arranjo" e "combinação", eu as baniria completamente
do ensino médio, assim como a notação C(8,5): elas são aliás
bem pouco usadas *fora* do ensino médio. A resposta do problema
acima seria descrita por quase qualquer matemático como binomial(8,5).

[]s, N.

=
Instruções para entrar na lista, sair da lista e usar a lista em
http://www.mat.puc-rio.br/~nicolau/olimp/obm-l.html
=
MSN Messenger: converse com os seus amigos online. Instale grátis. Clique aqui. 

=
Instruções para entrar na lista, sair da lista e usar a lista em
http://www.mat.puc-rio.br/~nicolau/olimp/obm-l.html
=


[obm-l] Problema dos soldados (Era: Livros)

2004-12-24 Por tôpico Antonio Neto

Ooops!! Falha nossa, esqueci de acrescentar *desde que A e B não sejam o mesmo*. Mas o problema faz sentido, o começo da solução é, imagine que A e B estejam na mesma linha. Então A é mais baixo, pois é o mais baixo da linha. Suponha agora que A e B estejam na mesma coluna. Novamente, A é mais baixo, pois B é o mais alto da coluna. E se não estiverem na mesma linha nem na mesma coluna? Vou deixar que pensem mais um pouco. Abraços, olavo.
From: "Daniel S. Braz" [EMAIL PROTECTED]
Reply-To: obm-l@mat.puc-rio.br
To: obm-l@mat.puc-rio.br
Subject: Re: [obm-l] Livros
Date: Thu, 23 Dec 2004 14:49:36 -0200

On Tue, 21 Dec 2004 08:40:23 +0000, Antonio Neto [EMAIL PROTECTED] wrote:
  Duzentos soldados estão arrumados em 10 linhas e 20 colunas. Em cada linha,
  escolha o soldado mais baixo, e entre estes 10 soldados escolha o mais alto.
  Chamemo-lo A. Agora eles voltam para os seus lugares e em cada coluna
  escolhemos o soldado mais alto, e entre estes 20 soldados escolhemos o mais
  baixo, chamando-o de B. Quem é mais alto, A ou B?

Utilizando uma matriz 3x6 ou inves de 10x20 (simplificando o exemplo)
e variando as alturas de 1 a 18 u.m (unidade de medida qualquer)

1 23456
7 89 10 11 12
13(ab) 14 15 16 17 18

a = b

8(b)15 2369
7 1317 16 121
5 4(a)14 18 10 11

a  b

de forma analoga conseguimos montar uma matriz onde b  a...então..ou
eu não entendi o prolema (bem provavel), ou ele nao faz sentido...

MSN Messenger: converse com os seus amigos online. Instale grátis. Clique aqui. 

=
Instruções para entrar na lista, sair da lista e usar a lista em
http://www.mat.puc-rio.br/~nicolau/olimp/obm-l.html
=


[obm-l] Livros

2004-12-21 Por tôpico Antonio Neto

Oi, Daniele,
Suponho que voce se refira a "The USSR Olympiad Problem Book", atualmente editado pela Dover, e que custa US$ 14,95 nos EUA. O ISBN é 0-486-27709-7. Este livro foi originalmente publicado em 1962 pela W. H. Freeman an Company, mas está esgotado. Realmente há QUASE o mesmo livro publicado pela MIR, chama-se "Selected Problems and Theorems of Elementary Mathematics", que aliás é o subtítulo do livro anterior, mesmos autores, mas não sei onde encontrá-lo. Alguém aqui na lista mandou um site onde comprar livros da MIR, anotei mas no momento não estou achando. Posso sugerir www.doverpublications.com, onde se encontram belíssimos livros de Matemática. Mas não esqueça de www.maa.org, o site da Mathematical Association of America, que tem coisas lindas. Em particular as séries Anneli Lax New Mathematical Library e 
a Dolciani Mathematical Expositions, que são um maná para professores, o que não sei se é o seu caso, mas é o meu. Se você mora no Rio ou São Paulo (eu sou do Rio) visite a MF, livraria que importa o que você quiser. Na rede, tente www.mf.com.br. 
Sem querer ser imodesto, tenho bastante informação sobre livros, pode escrever para mim por fora da lista, para não cairmos em off-topic. Se você me conhecesse, saberia que sou velho o bastante para querer que o Nicolau brigue comigo. E, para sair do off, vai um probleminha, como você pediu.
Duzentos soldados estão arrumados em 10 linhas e 20 colunas. Em cada linha, escolha o soldado mais baixo, e entre estes 10 soldados escolha o mais alto. Chamemo-lo A. Agora eles voltam para os seus lugares e em cada coluna escolhemos o soldado mais alto, e entre estes 20 soldados escolhemos o mais baixo, chamando-o de B. Quem é mais alto, A ou B?
Abraços a todos, olavo.
From: Daniela Yoshikawa [EMAIL PROTECTED]
Reply-To: [EMAIL PROTECTED]
To: [EMAIL PROTECTED]
Subject: [obm-l] Livros
Date: Mon, 20 Dec 2004 19:05:58 -0300 (ART)

Olá pessoal!

Onde posso encontrar o livro Contest Problem Book (não sei se é bem esse o nome)? Dizem que é um livro americano de competições matemáticas. Há um outro livro de matemática da Editora Mir também. Só que este não consigo achar em nenhum lugar.
Se possível, gostaria também que alguém me passasse alguns exercícios de ambos os livros caso tenha o conhecimento destes.

Desde já agradeço,
Abraços.

Daniele

MSN Messenger: converse com os seus amigos online. Instale grátis. Clique aqui. 

=
Instruções para entrar na lista, sair da lista e usar a lista em
http://www.mat.puc-rio.br/~nicolau/olimp/obm-l.html
=


Re: [obm-l] Material de Ensino Fundamental e Médio

2004-04-29 Por tôpico Antonio Neto



 
--- Augusto Cesar de Oliveira Morgado 
[EMAIL PROTECTED] escreveu:  Sou licenciado 
em Matemática e professor de 
  Matemática do Colégio Futuro, em Duque de Caxias, 
  RJ. 
 
eh isso ai..e mais isso aqui tb.. 
http://www.ensinomedio.impa.br/participantes/morgado.htm 
 


Eu ia responder, mas vi a sua resposta primeiro. Hah muito mais do que o Morgado declara, escondendo os seus titulos e empregos dos quais jah estah aposentado. Mas o primordial para mim, que tive a honra de ser aceito como amigo, eh a extraordinaria figurahumana daquele gordo adoravel. Abracos, olavo.MSN Messenger: converse com os seus amigos online. Instale grátis. Clique aqui. 
=
Instruções para entrar na lista, sair da lista e usar a lista em
http://www.mat.puc-rio.br/~nicolau/olimp/obm-l.html
=


RE: [obm-l] Provas IME 1996/1997-2003/2004

2004-04-19 Por tôpico Antonio Neto

Excelente idéia, Sergio. O Morgado ainda deve lembrar de quando me perguntou por telefone se eu tinha essas provas em formato digital, e eu não tinha, nem tenho. Obrigado, pelo menos por mim. Abracos, olavo.
From: [EMAIL PROTECTED] 
Reply-To: [EMAIL PROTECTED] 
To: [EMAIL PROTECTED] 
Subject: [obm-l] Provas IME 1996/1997-2003/2004 
Date: Fri, 16 Apr 2004 16:04:03 -0300 (BRT) 
 
Caros colegas da lista, 
Eu tenho um material sobre as provas de matematica do IME 
que eu gostaira de disponibilizar para todos. 
Eu tenho uma boa colecao de provas do IME, mas da' um trabalho 
gigantesco de coloca-las em formato eletronico. 
Motivado por esta lista eu comecei a fazer isto. 
Comecei pelas provas mais recentes. Estas sao mais 
faceis de achar na internet (propria homepage do ime 
ou em outros sites), mas achei que tinham mais a ver com o 
vestibular atual. A ideia e' ir incrementado aos poucos. 
(o ritmo depende das outras atividades, e' claro). 
MSN Messenger: converse com os seus amigos online. Instale grátis. Clique aqui. 
=
Instruções para entrar na lista, sair da lista e usar a lista em
http://www.mat.puc-rio.br/~nicolau/olimp/obm-l.html
=


[obm-l] Forma ção dos participantes da lista

2004-04-19 Por tôpico Antonio Neto

 Eu enrolo alunos do ensino médio. Esta eh uma preocupacao meio bizantin, hem??? Abracos, olavo.MSN Messenger: converse com os seus amigos online. Instale grátis. Clique aqui. 
=
Instruções para entrar na lista, sair da lista e usar a lista em
http://www.mat.puc-rio.br/~nicolau/olimp/obm-l.html
=


[obm-l] livros

2004-03-19 Por tôpico Antonio Neto

 Mas a livraria estah lah, pelo menois ateh sabado passado, estou no trabalho de novo, vou ver se acho um endereco eletronico diferente em casa, mas vale lembrar que o velho Graham Bell ainda nao virou um completo dinossauro... Abracos, olavo.
From: Nelson <[EMAIL PROTECTED]>
Reply-To: [EMAIL PROTECTED] 
To: [EMAIL PROTECTED] 
Subject: Re: [obm-l] livros 
Date: Thu, 18 Mar 2004 19:33:16 -0300 (ART) 
 
Olá Antônio, 
 
Por acaso a livraria é: 
Livraria Academia do Saber 
Av. Passos, 25 
Centro 
Tel: (21) 2242-4826 
Livros novos e usados 
http://www.aosaber.com.br/ 
 
Infelizmente, esse site está desativado. 
Espero estar errado... se você tiver o endereço eletrônico, eu agradeço. 
[]´s Nelson 
 
p.s.: Realmente, no site da livraria galileu consta os livros, mas eles acabaram de me avisar que eles estão na verdade esgotados. 
 
 
Antonio Neto <[EMAIL PROTECTED]>wrote: 
 
Por acaso, soube no sábado passado. Se voce estiver no Rio de Janeiro, encontrarah uma livraria na Av. Passos, 23 ou 25, mas estah de frente para a rua. Encontrei 5 ou 6 exemplares de cada, em bom estado. Sem querer imitar nenhum participante da lista, depois eu mando o endereco eletronico, eh que estou no trabalho. Abracos, olavo. 
 
 
 
 
 From: Nelson 
 Reply-To: [EMAIL PROTECTED] 
 To: [EMAIL PROTECTED] 
 Subject: [obm-l] livros 
 Date: Wed, 17 Mar 2004 11:36:14 -0300 (ART) 
  
 Olá a todos, 
  
 Alguém poderia me dizer como encontro os livros: 
 GEOMETRIA I e II, e ALGEBRA I (morgado, a.c., et alii) 
  
 E alguém sabe como entrar em contato com a editora Francisco Alves? 
  
 Desde já, agradeço. 
 Nelson 
 
 
 
 
 
 
 
 
- 
MSN Messenger: converse com os seus amigos online. Instale grátis. Clique aqui. = Instruções para entrar na lista, sair da lista e usar a lista em http://www.mat.puc-rio.br/~nicolau/olimp/obm-l.html = 
 
 
- 
Yahoo! Mail - O melhor e-mail do Brasil. Abra sua conta agora! 
MSN Messenger: converse com os seus amigos online. Instale grátis. Clique aqui. 
=
Instruções para entrar na lista, sair da lista e usar a lista em
http://www.mat.puc-rio.br/~nicolau/olimp/obm-l.html
=


[obm-l] livros

2004-03-18 Por tôpico Antonio Neto

 Por acaso, soube no sábado passado. Se voce estiver no Rio de Janeiro, encontrarah uma livraria na Av. Passos, 23 ou 25, mas estah de frente para a rua. Encontrei 5 ou 6 exemplares de cada, em bom estado. Sem querer imitar nenhum participante da lista, depois eu mando o endereco eletronico, eh que estou no trabalho. Abracos, olavo.
From: Nelson <[EMAIL PROTECTED]>
Reply-To: [EMAIL PROTECTED] 
To: [EMAIL PROTECTED] 
Subject: [obm-l] livros 
Date: Wed, 17 Mar 2004 11:36:14 -0300 (ART) 
 
Olá a todos, 
 
Alguém poderia me dizer como encontro os livros: 
GEOMETRIA I e II, e ALGEBRA I (morgado, a.c., et alii) 
 
E alguém sabe como entrar em contato com a editora Francisco Alves? 
 
Desde já, agradeço. 
Nelson 




 
MSN Messenger: converse com os seus amigos online. Instale grátis. Clique aqui. 
=
Instruções para entrar na lista, sair da lista e usar a lista em
http://www.mat.puc-rio.br/~nicolau/olimp/obm-l.html
=


[obm-l] Parâmetros curriculares nacionais

2004-01-30 Por tôpico Antonio Neto
  Tento ficar quieto no meu cannto, mas as vezes nao consigo, dada a 
burrice dos ministérios e dos supostos parametros. Ponha um ministro da 
educacao em uma sala de aula, por um ano, e ele poderah falar algo sobre 
ensino. No meu modo curto e grosso de ver, eh por ahi

On Thu, Jan 29, 2004 at 07:58:04PM -0200, Augusto Cesar de Oliveira Morgado 
wrote:
 De minha parte, declaro achar uma bobagem achar frações ordinárias uma
 bobagem.

Oi Morgado, você pode contar um pouco melhor para a gente que tem preguiça
de ler o documento do MEC como é esta coisa das frações ordinárias serem
ou não serem bobagem? Desculpe, eu sei que estou sendo bem folgado...
[]s, N.
_
MSN Messenger: instale grátis e converse com seus amigos. 
http://messenger.msn.com.br

=
Instruções para entrar na lista, sair da lista e usar a lista em
http://www.mat.puc-rio.br/~nicolau/olimp/obm-l.html
=


Re: [obm-l] Sobre um livro...

2003-10-12 Por tôpico Antonio Neto
  Recomendo veementemente. Se voce eh dado a emprestar livros, compre dois, 
pois ele serah roubado.  Abracos, olavo.

oi... ;-)

Gostaria de saber sobre um livro que me pareceu interessante...
Chama-se Challenging Mathematical Problems With Elementary Solutions, Vol.
I, de A. M. Yaglom e I. M. Yaglom...
Alguém o recomenda?
abraços a todos,
Alexandre
=
Instruções para entrar na lista, sair da lista e usar a lista em
http://www.mat.puc-rio.br/~nicolau/olimp/obm-l.html
=
_
MSN Messenger: instale grátis e converse com seus amigos. 
http://messenger.msn.com.br

=
Instruções para entrar na lista, sair da lista e usar a lista em
http://www.mat.puc-rio.br/~nicolau/olimp/obm-l.html
=


[obm-l] Reciprocas

2003-10-09 Por tôpico Antonio Neto
  Receio que a diferenca entre equacoes reciprocas e palindromas seja soh 
nomenclatura. Palindroma eh uma expressao que nao se altera quando lida da 
esquerda para a direita e vice-versa. Detalhe encantador, na parte mais 
ocidental da peninsula iberica nao hah palindromos, soh umas capicuas. 
Abracos, olavo.

_
MSN Messenger: instale grátis e converse com seus amigos. 
http://messenger.msn.com.br

=
Instruções para entrar na lista, sair da lista e usar a lista em
http://www.mat.puc-rio.br/~nicolau/olimp/obm-l.html
=


[obm-l] E o que fazer com eles?

2003-08-05 Por tôpico Antonio Neto
  Amigos, onde trabalho existe a educacao artistica, e a professora da 
quinta serie quer uma definicao para linha. Achei que as nossas definicoes 
usuais seriam meio transcendentes, e procuro auxilio dos colegas que atuam 
nesse segmento. Ou de quem possa me dizer algo razoavel. Obrigado, um 
abraco, olavo.

_
MSN Messenger: instale grátis e converse com seus amigos. 
http://messenger.msn.com.br

=
Instruções para entrar na lista, sair da lista e usar a lista em
http://www.mat.puc-rio.br/~nicolau/olimp/obm-l.html
=


[obm-l] Charlatanismo

2003-08-05 Por tôpico Antonio Neto
  Os amigos da lista já devem ter reparado que sou meio rarefeito em 
palavras, mas é apenas pelo fato de ter que fazer um monte de coisas e ter 
pouco tempo. Quando chego a uma questão da lista, alguém já respondeu, e 
evito redundâncias. Mas vamos abrir uma pequena exceção, sobre a referência 
a um ciadão que se apresenta como um mago da Matemática, fazendo qualquer um 
aprendê-la por meio de métodos mágicos e esotéricos. Talvez fosse bom para 
mim, que, utilizando apenas os métodos normais (entenda-se estudar), 
progredi menos do que gostaria. Mas acho isso uma notável empulhação. Uma 
ex-aluna fez-me a gentileza de enviar uma gravação de uma das entrevistas, 
não sei com que intuito, mas foi compensatório, pude tomar conhecimento. Mas 
acho que deveria trocar o subject dessa mensagem. Nem o autor original faria 
melhor escrevendo algo sobre COMO ENGANAR AS PESSOAS. Minha solidariedade 
(dispensável, apenas de amigo) ao Morgado, este, sim, um senhor um tanto 
acima do peso, mas que jamais enganou ninguém nem veio dizer que a 
Matemática está ao alcance de qualquer beócio. Perdão pelo off-topic, mas 
estava com essa fita engasgada, e não pude resistir. No mais, muitas das 
minhas decisões na vida não foram diferenciáveis, e é difícil abandonar a 
boca torta pelo hábito do cachimbo. Pois é, ainda por cima sou (tal qual o 
Morgado) um detestável tabagista. Abraços baforantes, de mim, olavo.

_
MSN Messenger: instale grátis e converse com seus amigos. 
http://messenger.msn.com.br

=
Instruções para entrar na lista, sair da lista e usar a lista em
http://www.mat.puc-rio.br/~nicolau/olimp/obm-l.html
=


RE: [obm-l] Duvidas

2003-06-13 Por tôpico Antonio Neto
  Caro Barone, V. não lembra de mim, mas voamos juntos para NY na Olimpíada 
Internacional de 1981, a do Nicolau. Obrigado pela resposta, mas a diferença 
de idade até que não é tão grande assim. Lembra que conosco foi o 
Pitombeira? Grande prazer em receber o seu e-mail, um abraço grande, olavo.


From: Angelo Barone Netto [EMAIL PROTECTED]
Reply-To: [EMAIL PROTECTED]
To: [EMAIL PROTECTED]
Subject: RE: [obm-l] Duvidas
Date: Sat, 7 Jun 2003 03:57:12 -0300 (EST)
Caro Olavo.
Eu devo ser mais velho do que V. e ainda uso o termo escolio
Aqui vai uma definicao nao matematica:
Escolio de uma demonstracao de um teorema e uma
proposicao que decorre (imediatamente) da demonstracao
mas nao decorre do enunciado do dito teorema.
Angelo Barone{\ --\ }Netto   Universidade de Sao Paulo
Departamento de Matematica Aplicada  Instituto de Matematica e Estatistica
Rua do Matao, 1010   Butanta - Cidade Universitaria
Caixa Postal 66 281  phone +55-11-3091-6162/6224/6136
05311-970 - Sao Paulo - SP   fax +55-11-3091-6131
Agencia Cidade de Sao Paulo
_
Tired of spam? Get advanced junk mail protection with MSN 8. 
http://join.msn.com/?page=features/junkmail

=
Instruções para entrar na lista, sair da lista e usar a lista em
http://www.mat.puc-rio.br/~nicolau/olimp/obm-l.html
=


RE: [obm-l] Duvidas

2003-06-04 Por tôpico Antonio Neto
  E haveria alguem da minha prisca geracao que lembrasse o que eh um 
escohlio? Abracos, olavo.


From: Leandro Lacorte Recôva [EMAIL PROTECTED]
Reply-To: [EMAIL PROTECTED]
To: [EMAIL PROTECTED]
Subject: RE: [obm-l] Duvidas
Date: Mon, 2 Jun 2003 16:29:53 -0700
Lemas sao pequenos teoremas que sao apresentados, geralmente, antes do
teorema principal a ser demonstrado. Portanto, para a demonstracao do
teorema nao ficar grande , as vezes e mais facil apresentar o lema antes
e depois somente utilizar os resultados dentro da demonstracao do
teorema principal.
Corolarios sao geralmente consequencias de um teorema previamente
demonstrado, pode ser um caso especial, etc...


-Original Message-
From: [EMAIL PROTECTED]
[mailto:[EMAIL PROTECTED] On Behalf Of Helder Suzuki
Sent: Monday, June 02, 2003 3:50 PM
To: [EMAIL PROTECTED]
Subject: [obm-l] Duvidas
Olá!

Eu sempre vejo em algumas demonstrações termos como
'lema', 'corolário' e etc...
O que significam?
Obrigado,
Helder T. Suzuki
_
STOP MORE SPAM with the new MSN 8 and get 2 months FREE* 
http://join.msn.com/?page=features/junkmail

=
Instruções para entrar na lista, sair da lista e usar a lista em
http://www.mat.puc-rio.br/~nicolau/olimp/obm-l.html
=


Re: [obm-l] AJUDA

2003-04-04 Por tôpico Antonio Neto
  Nao sei se estou atrasado e alguem ja respondeu, mas soh li hoje. O 
numero eh 142857, o periodo de 1/7. Isto ocorre sempre que 1/p, pprimo, tem 
um periodo com p-1 algarismos. Tais numeros sao chamados ciclicos. Posso 
mandar referencis bibliograficas, mas nao agora, estou longe da minha 
bisbilhoteca. Abracos, olavo.





From: A. C. Morgado [EMAIL PROTECTED]
Reply-To: [EMAIL PROTECTED]
To: [EMAIL PROTECTED]
Subject: Re: [obm-l] AJUDA
Date: Mon, 31 Mar 2003 22:14:36 -0300
1)
a=1 (se a1, 6N nao poderia ter a mesma quantidade de algarismos de N).
3N = bcdef1 (o 1=a so pode aparecer na ultima posiçao no 3N, pois o 5N nao 
pode terminar em 1 e os outros sao pares). Logo, N termina em 7, f=7.
2N termina em 4, 4N termina em 8 e 6N termina em 2, 5N termina em 5.
Os algarismos sao 1(inicial), 7(final), 4, 8, 2 e 5 (nao sei em que 
posiçoes)
S = 27
Se o problema tem soluçao, a soluçao eh 27.

Daniel Pini wrote:

OLá, alguem poderia me ajudar?
1-O número de seis algarismos N=abcdef é tal que quando multipicamos por 
2, 3, 4, 5, 6 obtemos números com os mesmos algarismos permutados 
ciclicamente. A soma dos alg. de N é: R:27




_
The new MSN 8: advanced junk mail protection and 2 months FREE* 
http://join.msn.com/?page=features/junkmail

=
Instruções para entrar na lista, sair da lista e usar a lista em
http://www.mat.puc-rio.br/~nicolau/olimp/obm-l.html
O administrador desta lista é [EMAIL PROTECTED]
=


Re: [obm-l] Máximos e Mínimos SEM DERIVADAS

2003-02-11 Por tôpico Antonio Neto
  Amigos,

  posso indicar um livro maravilhoso sobre o assunto: Maxima and Minima  
Without Calculus, do Ivan Niven, Dolciani Mathematical Expositions, numero 
6, The Mathematical Association of America, não sei se ainda estah em 
catalogo, foi comprado em 1981, na propria MAA, na companhia do Nicolau. 
Alô, Nicolau, não estou vendendo o livro, ele eh meu, eh soh uma indicacao. 
Dizer que ele eh otimo eh fazer pouco do Niven. Acessem www.maa.org. 
Abracos, olavo, RJ




From: Thyago Alexandre Kufner [EMAIL PROTECTED]
Reply-To: [EMAIL PROTECTED]
To: [EMAIL PROTECTED]
Subject: [obm-l] Máximos e Mínimos SEM DERIVADAS
Date: Wed, 5 Feb 2003 19:28:11 -0200

Olá colegas da lista

Recebi o seguinte exercício de um aluno:

Sendo x um nº positivo determine o menor valor de E= 5x + 16/x + 21

Normal, um exercício simples. Deriva, iguala a zero ...

Mas o que quero propor para a lista é o seguinte: tem como chegar ao
resultado SEM UTILIZAR CÁLCULO?

Proponho esta discussão por causa do seguinte artigo:

http://mathcircle.berkeley.edu/BMC4/Handouts/MaxMin.pdf

Aguardo resposta

Atenciosamente
Prof. Thyago
WebMaster cursinho.hpg.com.br



_
Tired of spam? Get advanced junk mail protection with MSN 8. 
http://join.msn.com/?page=features/junkmail

=
Instruções para entrar na lista, sair da lista e usar a lista em
http://www.mat.puc-rio.br/~nicolau/olimp/obm-l.html
O administrador desta lista é [EMAIL PROTECTED]
=


Re: [obm-l] Deve haver fissao da lista?

2003-02-11 Por tôpico Antonio Neto
  Amigo Nicolau,

  uma proposta brincalhona: vamos fazer uma fissão na lista. Em uma 
ficariamos todos, exceto dois. Na outra, o falso Wagner e o Faelccmm. Acho 
que tenho o apoio do Morgado. Abracos, olavo.







From: Nicolau C. Saldanha [EMAIL PROTECTED]
Reply-To: [EMAIL PROTECTED]
To: [EMAIL PROTECTED]
Subject: [obm-l] Deve haver fissao da lista?
Date: Tue, 11 Feb 2003 14:46:37 -0200

Esta proposta de fissão da lista obm-l já apareceu várias vezes.
Tecnicamente ela é muito fácil de ser implementada, a pergunta
é se tal fissão é desejável. Todas as vezes que a proposta foi feita
houve um pouco de discussão e me parecia no final que não havia maioria
a favor da fissão.

O fato da discussão já ter ocorrido antes não é, a meu ver, motivo
para que a discussão não volte a ocorrer pois a realidade da lista
tem mudado bastante. Por exemplo, o número de mensagens tem crescido
muito. Há dois anos eu era totalmente contra a fissão: hoje não tenho
mais tanta certeza...

Eu sugiro que quem tiver uma opinião mande um e-mail particular para
mim dizendo se é a favor da fissão e porque. Dentro de uma semana
eu mandarei o resultado da votação para a lista.

[]s, N.
=
Instruções para entrar na lista, sair da lista e usar a lista em
http://www.mat.puc-rio.br/~nicolau/olimp/obm-l.html
O administrador desta lista é [EMAIL PROTECTED]
=



_
Add photos to your messages with MSN 8. Get 2 months FREE*. 
http://join.msn.com/?page=features/featuredemail

=
Instruções para entrar na lista, sair da lista e usar a lista em
http://www.mat.puc-rio.br/~nicolau/olimp/obm-l.html
O administrador desta lista é [EMAIL PROTECTED]
=


Re: [obm-l] RE: [obm-l] classifiquem a função

2003-01-14 Por tôpico Antonio Neto
  Do alto das minhas venerandas barbas branquinhas, endosso as 
homograficas. Abracos, olavo.






From: A. C. Morgado [EMAIL PROTECTED]
Reply-To: [EMAIL PROTECTED]
To: [EMAIL PROTECTED]
Subject: Re: [obm-l] RE: [obm-l] classifiquem a função
Date: Sun, 12 Jan 2003 22:08:53 -0200

Tirando uma palavra do ostracismo:
essas funçoes que sao quocientes de polinomios de primeiro grau  ( de 
raizes diferentes ) tem como graficos hiperboles equilateras com uma 
assintota horizontal e outra vertical  sao (ou eram, ha muito tempo nao 
vejo ninguem uar esse termo) chamadas (pelo menos pelos franceses) de 
HOMOGRAFICAS.
Morgado

Alguem ha muito tempo escreveu:

Como vocês classificariam a seguinte função:
f(x) = x-a / bx+a. Nos livros de 2º grau temos a função afim ou do 1ºgrau 
com a expressão ax+b. Temos a função quadrática com a expressão ax^2+bx+c 
etc. E na expressão da função acima como poderiamos classificar ? Eu acho 
que é do 1º grau pois temos somente a variável x com expoente unitário, 
mas e quanto ao gráfico?





_
MSN 8: advanced junk mail protection and 2 months FREE*. 
http://join.msn.com/?page=features/junkmail

=
Instruções para entrar na lista, sair da lista e usar a lista em
http://www.mat.puc-rio.br/~nicolau/olimp/obm-l.html
O administrador desta lista é [EMAIL PROTECTED]
=


Re: [obm-l] onde comprar livros

2002-12-23 Por tôpico Antonio Neto
  Olha, *Matematica divertida e delirante*, que por acaso tenho, é de Malba 
Tahan, edicao antiquerrima, quase de minha provecta idade. Vai ser dificil 
achar, e a editora nao sei, estou no trabalho. Acho que nao ajudou nada, mas 
a solucao é mesmo um bom sebo e muita sorte. Abracos, olavo, RJ






From: Jose Francisco Guimaraes Costa [EMAIL PROTECTED]
Reply-To: [EMAIL PROTECTED]
To: obm-l [EMAIL PROTECTED]
Subject: [obm-l] onde comprar livros
Date: Sat, 21 Dec 2002 23:08:51 -0400

Um componente de um outro grupo do qual participo está pedindo ajuda para 
comprar os livros abaixo, ambos da Editora Saraiva:

- Problemas famosos e curiosos da matematica

- Matematica divertida e delirante

Ele mora em Manaus e já tentou sebos virtuais e grandes livrarias, sem 
sucesso.

Alguém poderia ajudá-lo?

JF (Rio)



_
Protect your PC - get McAfee.com VirusScan Online 
http://clinic.mcafee.com/clinic/ibuy/campaign.asp?cid=3963

=
Instruções para entrar na lista, sair da lista e usar a lista em
http://www.mat.puc-rio.br/~nicolau/olimp/obm-l.html
O administrador desta lista é [EMAIL PROTECTED]
=


Re: [obm-l] Continuo Perguntando...

2002-09-10 Por tôpico Antonio Neto




From: e isso mesmo [EMAIL PROTECTED]
Reply-To: [EMAIL PROTECTED]
To: [EMAIL PROTECTED]
Subject: [obm-l] Continuo Perguntando...
Date: Tue, 10 Sep 2002 14:57:34 -0300




Amigos, alguém poderia me explicar detalhadamente o que são números 
randômicos e como se constrói esses números? Li isso em algum lugar sobre 
loterias.
Obrigado


   Randomico eh ingles de Praça Mauah (sou do Rio de Janeiro), onde hah 
senhoritas que servem de guias aos marinheiros americanos, de onde tambem 
veio *rango*, de *hungry*. Em ingles, *at random* quer dizer ao acaso, e 
numeros randomicos sao numeros aleatorios. Abracos, olavo.

_
Chat with friends online, try MSN Messenger: http://messenger.msn.com

=
Instruções para entrar na lista, sair da lista e usar a lista em
http://www.mat.puc-rio.br/~nicolau/olimp/obm-l.html
O administrador desta lista é [EMAIL PROTECTED]
=



Re: [obm-l] CRUEL

2002-08-15 Por tôpico Antonio Neto

   Neste tipo de problemas, costuma-se evitar acidentes de percurso, supondo 
que as coisas sao como no mundo de Candido, o melhor dos mundos possiveis 
(leiam Voltaire). Costuma-se pedir o numero maximo. Abracos, olavo.


From: Eduardo Casagrande Stabel [EMAIL PROTECTED]
Reply-To: [EMAIL PROTECTED]
To: [EMAIL PROTECTED]
Subject: Re: [obm-l] CRUEL
Date: Wed, 14 Aug 2002 17:17:23 -0300

From: Bruno F. C. Leite [EMAIL PROTECTED]
  At 12:21 14/08/02 -0400, you wrote:
  Num polígono convexo de n lados, quando se constrói todas as diagonais
  aparecem  pontos de interseção entre as diagonais. Determinar o número 
de
  pontos de interseção?
 
  Vamos supor que não há duas diagonais paralelas.
 
  Note que a cada ponto de intersecção podemos associar as duas diagonais 
ou
  o quadrilátero formado pelos extremos destas diagonais. Logo há uma
bijeção
  entre o número de intersecções e o de quadriláteros com vértices 
contidos
  no conjunto de vértices do poligono...logo a resposta é binomial(n,4).
 
  Está certo?
 
  Bruno Leite
  http://www.ime.usp.br/~brleite

Mas e se dois quadriláteros distintos tiverem o mesmo ponto de interseção
das suas diagonais? Ou isso nunca ocorre por que nenhuma das diagonais é
paralelas, por hipótese sua?

=
Instruções para entrar na lista, sair da lista e usar a lista em
http://www.mat.puc-rio.br/~nicolau/olimp/obm-l.html
O administrador desta lista é [EMAIL PROTECTED]
=




_
Join the world’s largest e-mail service with MSN Hotmail. 
http://www.hotmail.com

=
Instruções para entrar na lista, sair da lista e usar a lista em
http://www.mat.puc-rio.br/~nicolau/olimp/obm-l.html
O administrador desta lista é [EMAIL PROTECTED]
=



Re: [obm-l] Questão interessante.

2002-08-14 Por tôpico Antonio Neto

   Considere o cidadao na intersecao da linha do mais baixo dos altinhos com 
a coluna do mais alto dos baixinhos. Abracos, olavo.


From: Jose Francisco Guimaraes Costa [EMAIL PROTECTED]
Reply-To: [EMAIL PROTECTED]
To: obm-l [EMAIL PROTECTED]
Subject: [obm-l] Questão interessante.
Date: Tue, 13 Aug 2002 15:42:25 -0300

Não estou conseguindo partir. Tentando resolver no braço - afinal de 
contas,
para que existem computadores? - estou achando que o mais baixo entre os
mais altos das suas colunas é também o mais alto entre os mais baixos das
suas linhas. Dá para fornecer uma um ponto de partida?

JF

-Mensagem Original-
De: Augusto Cesar de Oliveira Morgado [EMAIL PROTECTED]
Para: [EMAIL PROTECTED]
Enviada em: Quinta-feira, 8 de Agosto de 2002 11:06
Assunto: Re: [obm-l] Questão interessante.


  Na verdade, o problema é russo e de data anterior a 1966. Mas é muito
bonito.
  Morgado
 
 
  Em Wed, 7 Aug 2002 22:13:01 -0300, Eduardo Casagrande Stabel
[EMAIL PROTECTED] disse:
 
   Olá pessoal!
  
   Compartilho com vocês esta questão que, tenho certeza, todos vão 
adorar.
  
   (Inglaterra - 1966) Cem pessoas de diferentes alturas são acomodadas 
num
   grande tabuleiro 10 x 10. O indivíduo X, o mais baixo dentre as 10
pessoas
   mais altas em suas colunas, mede uma altura diferente do indivíduo Y, 
o
mais
   alto dentro as 10 pessoas mais baixas em suas linhas. Quem é mais 
baixo:
X
   ou Y?
  
   Eduardo.
  
  
=
   Instruções para entrar na lista, sair da lista e usar a lista em
   http://www.mat.puc-rio.br/~nicolau/olimp/obm-l.html
   O administrador desta lista é [EMAIL PROTECTED]
  
=
  
  
 
  
=
  Instruções para entrar na lista, sair da lista e usar a lista em
  http://www.mat.puc-rio.br/~nicolau/olimp/obm-l.html
  O administrador desta lista é [EMAIL PROTECTED]
  
=
 

=
Instruções para entrar na lista, sair da lista e usar a lista em
http://www.mat.puc-rio.br/~nicolau/olimp/obm-l.html
O administrador desta lista é [EMAIL PROTECTED]
=




_
Send and receive Hotmail on your mobile device: http://mobile.msn.com

=
Instruções para entrar na lista, sair da lista e usar a lista em
http://www.mat.puc-rio.br/~nicolau/olimp/obm-l.html
O administrador desta lista é [EMAIL PROTECTED]
=



[obm-l] Infinitos

2002-07-02 Por tôpico Antonio Neto

   Existe tradução, com o nome Teoria Ingênua dos Conjuntos, do tempo em 
que editora tinha acento circunflexo, o tradutor foi o Irineu Bicudo, Ed. 
Polígono, com a colaboração da Ed. da USP. O consultor de Matemática era um 
certo Jacy Monteiro. Velhinho também é cultura :-)...


Existem muitos conceitos de infinito.
Um dos mais importantes é o conceito de cardinais infinitos (de Cantor)
sobre o qual já se falou muito nesta lista (veja os arquivos) e sobre
o qual você pode ler também em Naïve Set Theory, de Halmos (acho que
existe tradução).

No seu exemplo o cardinal é o mesmo, a função

f: [0,1]  - [0,10]
  x   |-  10x

é uma bijeção.

[]s, N.
=
Instruções para entrar na lista, sair da lista e usar a lista em
http://www.mat.puc-rio.br/~nicolau/olimp/obm-l.html
O administrador desta lista é [EMAIL PROTECTED]
=




_
Chat with friends online, try MSN Messenger: http://messenger.msn.com

=
Instruções para entrar na lista, sair da lista e usar a lista em
http://www.mat.puc-rio.br/~nicolau/olimp/obm-l.html
O administrador desta lista é [EMAIL PROTECTED]
=



Re: [obm-l] 3 circulos!

2002-06-18 Por tôpico Antonio Neto

   Aproveita e demonstra que r + r -1 +r_2 = h, onde h é a altura relativa a 
hipotenusa. Abracos, olavo.


From: Rafael WC [EMAIL PROTECTED]
Reply-To: [EMAIL PROTECTED]
To: OBM [EMAIL PROTECTED]
Subject: [obm-l] 3 circulos!
Date: Sun, 16 Jun 2002 21:47:30 -0700 (PDT)

Oi Pessoal!

Alguém conseguiria me dar uma indica do que usar nesse
exercício?

Os raios dos circulos inscritos num triangulo
retangulo ABC e nos dois triangulos ABH e ACH
determinados pela altura relativa à hipotenusa BC são
respectivamente r ,r1 e r2. Demonstrar que:
r² = r1² + r2²

Valeu!

Rafael.

=
Rafael Werneck Cinoto
ICQ# 107011599
  [EMAIL PROTECTED]
[EMAIL PROTECTED]
http://www.rwcinoto.hpg.com.br/

__
Do You Yahoo!?
Yahoo! - Official partner of 2002 FIFA World Cup
http://fifaworldcup.yahoo.com
=
Instruções para entrar na lista, sair da lista e usar a lista em
http://www.mat.puc-rio.br/~nicolau/olimp/obm-l.html
O administrador desta lista é [EMAIL PROTECTED]
=


_
Join the world’s largest e-mail service with MSN Hotmail. 
http://www.hotmail.com

=
Instruções para entrar na lista, sair da lista e usar a lista em
http://www.mat.puc-rio.br/~nicolau/olimp/obm-l.html
O administrador desta lista é [EMAIL PROTECTED]
=



Re: [obm-l] t. dos nºs

2002-06-11 Por tôpico Antonio Neto

   Lindissimo livro!!! Abracos, olavo.


From: Salvador Addas Zanata [EMAIL PROTECTED]
Reply-To: [EMAIL PROTECTED]
To: [EMAIL PROTECTED]
Subject: Re: [obm-l] t. dos nºs
Date: Tue, 11 Jun 2002 10:50:25 -0300 (EST)



O primeiro problema so pode ter solucao se p=4n+1.

Para ver isso, observe que a deve ser par e b impar. Logo a^2+b^2 e da
forma: 4c^2+4d^2+4c+1, que e da forma 4n+1.

De fato todo primo da forma 4n+1 se escreve de um unico jeito como a soma
de 2 quadrados. Tem um livro chamado 100 great elementary problems: Their
history and solutions Heinrich Dorrie, que tem essa prova e muitas outras
bacanas. Alias esse livro apresenta as melhores provas de cada
problema. E da Dover e nao e dificil de achar.


Abraco,

Salvador


On Tue, 11 Jun 2002, Adherbal Rocha Filho wrote:

 
  ajuda:
 
  Mostrar q se o primo p é tal q p==3(mod4), então a equação p^2= a^2 +b^2
  possui solução inteira
 
  mostre q todo quadrado perfeito pode ser representado como soma dos
  quadrados de racionais ,naum inteiros, r e s.
 
  valeu!
 
  _
  Chegou o novo MSN Explorer. Instale já. É gratuito:
  http://explorer.msn.com.br
 
  
=
  Instruções para entrar na lista, sair da lista e usar a lista em
  http://www.mat.puc-rio.br/~nicolau/olimp/obm-l.html
  O administrador desta lista é [EMAIL PROTECTED]
  
=
 

=
Instruções para entrar na lista, sair da lista e usar a lista em
http://www.mat.puc-rio.br/~nicolau/olimp/obm-l.html
O administrador desta lista é [EMAIL PROTECTED]
=




_
Chat with friends online, try MSN Messenger: http://messenger.msn.com

=
Instruções para entrar na lista, sair da lista e usar a lista em
http://www.mat.puc-rio.br/~nicolau/olimp/obm-l.html
O administrador desta lista é [EMAIL PROTECTED]
=



Re: [obm-l] Re: Maio01

2002-05-11 Por tôpico Antonio Neto




   Talvez eu não tenha sido claro o suficiente em alguns pontos pq estou 
com sono agora, mas os resultados que eu não msotrei são facilmente 
comprováveis - vide a relação eentre o lado do polígono regular e o raio da 
circunscrita. O único resultado que eu usei e não sei mostrar é o valor do 
sen(18). Esse eu realmente colei  :)))
Alexandre Tessarollo

Sem figura eh mais dificil, mas trace um triangulo isosceles de lados 1, 1 e 
x, e angulos de 36, 72 e 72. tracando a bissetriz de um dos angulos de 72, 
vc encontra outro triangulo semelhante ao primeiro, e acha x por semelhanca. 
Dahi, a bissetriz do angulo de 36 intercepta x no ponto medio, e... Abracos, 
olavo

_
Get your FREE download of MSN Explorer at http://explorer.msn.com/intl.asp.

=
Instruções para entrar na lista, sair da lista e usar a lista em
http://www.mat.puc-rio.br/~nicolau/olimp/obm-l.html
O administrador desta lista é [EMAIL PROTECTED]
=



Re: [obm-l] Soma de fatoriais

2002-04-01 Por tôpico Antonio Neto

   Vajavamos eu e o Morgado na semana santa. Fui a Fortaleza. Ainda nao li 
tudo, alguem deve ter reparado, mas dah pra fazer por 10!. Abracos, olavo


From: Augusto César Morgado [EMAIL PROTECTED]
Reply-To: [EMAIL PROTECTED]
To: [EMAIL PROTECTED]
Subject: Re: [obm-l] Soma de fatoriais
Date: Sat, 30 Mar 2002 10:05:40 -0300

De 20! para cima todos os fatoriais são múltiplos de 100, pois contêm os
fatores 20 e 5 


_
MSN Photos is the easiest way to share and print your photos: 
http://photos.msn.com/support/worldwide.aspx

=
Instruções para entrar na lista, sair da lista e usar a lista em
http://www.mat.puc-rio.br/~nicolau/olimp/obm-l.html
O administrador desta lista é [EMAIL PROTECTED]
=



[obm-l] Prato cheio.

2002-03-24 Por tôpico Antonio Neto

   Esse grupo de amantes dos complexos está crescendo. Outro dia eu resolvia 
um exercicio que pedia a diferenca entre a maior e a menor raizes de uma 
equacao do quarto grau que tinha duas raizes complexas, e um aluno perguntou 
qual das complexas era a maior. Que prato cheio para falar de relacoes de 
ordem, hein??? Ele conseguiu mudar totalmente a aula... Abracos, olavo.


From: Jose Paulo Carneiro [EMAIL PROTECTED]
Reply-To: [EMAIL PROTECTED]
To: [EMAIL PROTECTED]
Subject: Re: [obm-l] integral impropria
Date: Sun, 24 Mar 2002 20:53:40 -0300

Em primeiro lugar, parabens pelo uso de complexos (atencao, Olavo! mais
um!).
A solucao do Marcio estah correta (e tambem nao conheco melhor),
mas acho melhor substituir a frase o que voce pede eh a parte real de
I=... por
considere a integral I(R)=
JP


_
Get your FREE download of MSN Explorer at http://explorer.msn.com/intl.asp.

=
Instruções para entrar na lista, sair da lista e usar a lista em
http://www.mat.puc-rio.br/~nicolau/olimp/obm-l.html
O administrador desta lista é [EMAIL PROTECTED]
=



Re: [obm-l] álgbra

2002-03-22 Por tôpico Antonio Neto

   Olha aí uma grande oportunidade para JP usar seu entusismo pelos 
complexos. Interprete a^2 + b^2 como o quadrado do módulo de a + bi. 
Abracos, olavo.


From: Rafael WC [EMAIL PROTECTED]
Reply-To: [EMAIL PROTECTED]
To: [EMAIL PROTECTED]
Subject: [obm-l] álgbra
Date: Thu, 21 Mar 2002 12:42:41 -0800 (PST)

Pessoal, já estou ficando louco com essa questão, veja
se alguém consegue resolver. Eu já teho as respostas,
mas mesmo assim não consegui uma resolução:
se (5² + 9²)(12² + 17²) for escrito sob a forma a² +
b² então a + b é igual a ?
resp: 236 ou 286 (213² + 23² ou 193² + 93²)

Obrigado,

Rafael.

=
Rafael Werneck Cinoto
   ICQ# 107011599
 [EMAIL PROTECTED]
   [EMAIL PROTECTED]
   [EMAIL PROTECTED]
http://www.rwcinoto.hpg.com.br/

__
Do You Yahoo!?
Yahoo! Movies - coverage of the 74th Academy Awards®
http://movies.yahoo.com/
=
Instruções para entrar na lista, sair da lista e usar a lista em
http://www.mat.puc-rio.br/~nicolau/olimp/obm-l.html
O administrador desta lista é [EMAIL PROTECTED]
=


_
Chat with friends online, try MSN Messenger: http://messenger.msn.com

=
Instruções para entrar na lista, sair da lista e usar a lista em
http://www.mat.puc-rio.br/~nicolau/olimp/obm-l.html
O administrador desta lista é [EMAIL PROTECTED]
=



Re: [obm-l] Continuação II Triângulos (livro A.C. Morgado)

2002-03-19 Por tôpico Antonio Neto

   Soh uma observação, o Morgado, bem como o Wagner e o Miguel Jorge nao sao 
tao saudosos assim, foram recentemente vistos em lugares bem terrenos, com 
atividades nada angelicais... Abracos, olavo.


_
Get your FREE download of MSN Explorer at http://explorer.msn.com/intl.asp.

=
Instruções para entrar na lista, sair da lista e usar a lista em
http://www.mat.puc-rio.br/~nicolau/olimp/obm-l.html
O administrador desta lista é [EMAIL PROTECTED]
=



[obm-l] Teorema dos Quatro Quadrados de Lagrange

2002-02-21 Por tôpico Antonio Neto

   Oi, Jose,

   corroborando o Nicolau, o livro eh excelente mesmo, se voce morar no Rio 
pode escanear aqui em casa. Abracos, olavo.


From: Nicolau C. Saldanha [EMAIL PROTECTED]
Reply-To: [EMAIL PROTECTED]
To: [EMAIL PROTECTED]
Subject: Re: [obm-l] Teorema dos Quatro Quadrados de Lagrange
Date: Thu, 21 Feb 2002 16:28:47 -0200

On Thu, Feb 21, 2002 at 02:07:54PM -0300, Jose Francisco Guimaraes Costa 
wrote:
  Alguém saberia me dizer onde posso encontrar a demonstração do Teorema 
dos
  Quatro Quadrados de Lagrange? Ou, pelo menos, a linha básica seguida na
  demonstração?

Uma referência é o capítulo 20 de An Introduction to the Theory of 
Numbers,
de Hardy and Wright. Aliás um ótimo livro, recomendo a compra a quem tiver
interesse pelo assunto. []s, N.
=
Instruções para entrar na lista, sair da lista e usar a lista em
http://www.mat.puc-rio.br/~nicolau/olimp/obm-l.html
O administrador desta lista é [EMAIL PROTECTED]
=




_
Chat with friends online, try MSN Messenger: http://messenger.msn.com

=
Instruções para entrar na lista, sair da lista e usar a lista em
http://www.mat.puc-rio.br/~nicolau/olimp/obm-l.html
O administrador desta lista é [EMAIL PROTECTED]
=



Re: [obm-l] En: Livros interessantes

2002-02-07 Por tôpico Antonio Neto

   Oi, Rogerio,

   experimenta o *Introducao a Historia da Matematica*, do Howard Eves, Ed 
da Unicamp. Ja foi citado aqui na lista. Gosto de situar meus alunos, e 
rinha o Boyer, mas ele estava no meu carro qdo fui assaltado, e os ladroes 
deviam gostar do assunto, pois nem o livro sobrou. Fui a Bienal do Livro e 
comprei esse, ainda estou pelo meio, mas achei otimo, principalmente pelos 
exercicios.

   Complementando um e-mail anterior, nada para deslumbrar uns aluninhos 
como qualquer coisa de um cidadão chamado Martin Gardner, no catalogo da 
Dover ha muita coisa dele. Prometo, qdo tiver um tempinho, mandar uma 
pequena lista de coisas aqui da minha modesta bisbilhoteca. Abracos, olavo, 
Rio de Janeiro.


From: Rogerio Fajardo [EMAIL PROTECTED]
Reply-To: [EMAIL PROTECTED]
To: [EMAIL PROTECTED]
Subject: Re: [obm-l] En: Livros interessantes
Date: Fri, 08 Feb 2002 01:28:36 +

Olá, colegas

Em falar em livros interessantes, eu gostaria de saber se alguém conhecem
bons livros de história da matemática. Mas gostaria de livros que não
enfatizassem a história, em si, mas a matemática. Explicando melhor: eu
queria conhecer melhor a matemática em seu contexto histórico, o que 
motivou
a criação de cada área da mtemática, por que sentiram a necessidade de 
criar
a lógica matemática como fundamento da matemática, por que criaram a
topologia, a teoria dos conjuntos, o ZFC, etc. Às vezes sinto falta dessas
notas históricas para apreciar e entender melhor os diversos ramos da
matemática.

Rogério


_
Get your FREE download of MSN Explorer at http://explorer.msn.com/intl.asp.

=
Instruções para entrar na lista, sair da lista e usar a lista em
http://www.mat.puc-rio.br/~nicolau/olimp/obm-l.html
O administrador desta lista é [EMAIL PROTECTED]
=



Re: [obm-l] corredor

2002-01-26 Por tôpico Antonio Neto

   Receio que voce tenha esquecido de especificar o estado inicial dos 
armários. O usual é todos estarem fechados no inicio. Note que só ficam 
abertos aqueles que forem mexidos um número ímpar de vezes, isso ajuda? 
Abracos, olavo, rio de janeiro.


From: pichurin [EMAIL PROTECTED]
Reply-To: [EMAIL PROTECTED]
To: [EMAIL PROTECTED]
Subject: [obm-l] corredor
Date: Fri, 25 Jan 2002 23:53:50 -0300 (ART)

Em um corredoe existem 900 armários numerados de 1 a
900.Novecentas pessoas numeradas de 1 a 900 atravessam
este corredor ,uma a uma, em ordem crescente de
numeração.Cada pessoa deve reverter os armários que
sAõ múltiplos de sua numeração.Por exemplo, a pessoa
de número 4 deve mexer nor armários 4,8,12,16,20,etc,
abrindo aqyeles que estÃo fechados e fechando aqueles
que estão abertos.Ao final, quais armários estarão
abertos e quais estarão fechados?

___
Yahoo! GeoCities
Tenha seu lugar na Web. Construa hoje mesmo sua home page no Yahoo! 
GeoCities. É fácil e grátis!
http://br.geocities.yahoo.com/
=
Instruções para entrar na lista, sair da lista e usar a lista em
http://www.mat.puc-rio.br/~nicolau/olimp/obm-l.html
O administrador desta lista é [EMAIL PROTECTED]
=






_
Send and receive Hotmail on your mobile device: http://mobile.msn.com

=
Instruções para entrar na lista, sair da lista e usar a lista em
http://www.mat.puc-rio.br/~nicolau/olimp/obm-l.html
O administrador desta lista é [EMAIL PROTECTED]
=



Re: Exercícios

2002-01-16 Por tôpico Antonio Neto

   Desculpem, mas eh claro que existe o circulo. Dados 3 pontos nao 
colineares, sempre existe um circulo que os contem. O centro deste está no 
ponto médio do segmento que liga os médios de AB e AD. Abracos, olavo


From: Ricardo Miranda [EMAIL PROTECTED]
Reply-To: [EMAIL PROTECTED]
To: [EMAIL PROTECTED]
Subject: Re: Exercícios
Date: Tue, 15 Jan 2002 01:25:18 -0200


-Mensagem original-
De: [EMAIL PROTECTED] [EMAIL PROTECTED]
Para: [EMAIL PROTECTED] [EMAIL PROTECTED]
Data: Terça-feira, 15 de Janeiro de 2002 00:14
Assunto: Exercícios


 1)Considere o círculo que passa pelo vértice A de um quadrado ABCD e 
pelos
 pontos médios dos lados AB e AD. Sendo L a medida do lado do quadrado, o
 segmento determinado pela tangente a esse círculo traçada por C tem
comprimento:


Tem certeza q é um quadrado?
De modo algum to conseguindo fazer o tal círculo passar por A e pelos 
pontos
médios de AB e AD.
Se ele passa por A, nao passa por médios de AB e AD.. Se vc considerar o A 
o
centro do círculo sim,
ele passa pelos pontos médios (desde que AB = r/2).

[]s
Ricardo Miranda





_
Get your FREE download of MSN Explorer at http://explorer.msn.com/intl.asp.




Re: beal

2001-12-18 Por tôpico Antonio Neto

   Oi, Gabriel, não sei se ha capitulos publicados, o Luis Lopes, que é o 
autor, deve saber. Mas o livro eh muito bom, se vc reside no Rio, pode 
encontrar na Interciencia. Abracos, olavo.


From: gabriel guedes [EMAIL PROTECTED]
Reply-To: [EMAIL PROTECTED]
To: [EMAIL PROTECTED]
Subject: Re: beal
Date: Mon, 17 Dec 2001 20:20:49 -0200

Oi Olavo e demais colegas,
se não me engano  es te livro tem algums capitulos  postos num site alguem
tem o endereço deste???
- Original Message -


_
MSN Photos is the easiest way to share and print your photos: 
http://photos.msn.com/support/worldwide.aspx




Re: beal

2001-12-17 Por tôpico Antonio Neto

   Tenta o *Manual de Inducao Finita*, do Luis, que participa da lista. 
Excelente. Abracos, olavo.


From: gabriel guedes [EMAIL PROTECTED]
Reply-To: [EMAIL PROTECTED]
To: [EMAIL PROTECTED]
Subject: beal
Date: Sat, 15 Dec 2001 18:46:37 -0200

tudo bem colegas da lista,
1)Alguem ja ouviu  falar na conjectura de beal oque que ela propõe e etc???

2)Estava dando uma olhada em indução finita , e queria me a profundar 
,alguem conhece um bom livro  ?


_
Get your FREE download of MSN Explorer at http://explorer.msn.com/intl.asp.




Re: Topologia

2001-04-19 Por tôpico Antonio Neto


  Um problema que comumente dizem ser de topologia é o famoso problemas 
das
  sete pontes.

Proposto pelo genial Henri Poincaré! http://yakumo72.tripod.com/ eh meu 
site
totalmente dedicado ao Poincaré.


   Receio que haja um entusiasmo exagerado nesta afirmativa. Nao quanto a 
genialidade de Poincare, mas a autoria do problema. Estou citando de 
memoria, pois estou no trabalho, mas as sete pontes ficavam em Koenigsberg, 
hoje Kaliningrado, e ao que eu saiba era um problema popular saber se o 
passeio podia ser realizado. Foi primeiramente resolvido por Euler, ate onde 
estou informado. Agradeço correcoes a alguma besteira. Abracos, olavo.
_
Get Your Private, Free E-mail from MSN Hotmail at http://www.hotmail.com.




  1   2   >